30 Brain Teaser Interview Questions & Detailed Answers

If you have learned about consulting interviews, the term "Brain Teasers" is certainly familiar to you at some stage, however, are recruiters really using it in interviews or is this just a myth? According to a senior consultant/interviewer of McK and BCG, the use of brainteasers differs across the tiers:

  • Not used in MBB
  • Sometimes used in Tier 2 firms (Roland Berger, Oliver Wyman, etc.)
  • Frequently used in Tier 3 firms (Arthur D. Little, etc.)

Brain Teasers are now being used less in interviews by consultants since they have recognized the limitations of brain teasers as an assessment tool and have shifted their focus to more practical case interviews that reflect the actual work of a consultant. 

However, not all consulting businesses share the same opinion. In fact, many consulting firms still utilize brainteasers to seek out candidates who are capable of thinking rationally and creatively under pressure. Hence, to best prepare for the interviews, you should learn something about this subject.

Table of Contents

What are Brain Teasers in Consulting Interviews?

Brain teasers are “trick questions”.

Brain teasers are puzzles or problems that require a candidate to use logic, creativity, and critical thinking to arrive at a solution . These quizzes come in various forms, such as math problems, logic puzzles, or situational challenges and also can be applied in a wide range of cases in different fields. 

Brain teasers aim to test the ability of people to think outside the box, analyze complex situations, and solve problems under pressure.

Brain teasers used to be important

According to former consultant of Accenture , brain teasers were used to be an important part of consulting interviews as they are not placed in a business setting so it can provide the company with insights regarding how applicants apply logic and creative thinking to solve problems outside of expertise areas. 

This can be necessary when assessing freshmen without much work experience, hence, it also allowed the employer to see how the candidates responded under pressure when confronted with a novel problem

Another reason is they provide a means of testing a candidate's analytical and problem-solving skills which is a critical skill for a consultant as they are often called upon to solve difficult business problems for their clients. They often involve a complex scenario or ambiguous problem that requires candidates to think outside the box or use data to make informed decisions. 

Additionally, brain teasers can also provide insights into a candidate's communication skills . Consulting firms tend to look for candidates who can not only solve complex problems but also communicate their thought process and reasoning successfully. 

As consultants must be able to convey their ideas to customers, coworkers, and stakeholders so the capacity to explain a solution succinctly and clearly is a requirement.

Examples of brain teasers used in consulting interviews

Examples of brain teasers used in consulting interviews vary in complexity and can range from simple math problems to more intricate logic puzzles. 

One example of a brain teaser is the "Three Light Bulbs Problem." In this scenario, a candidate is presented with a room containing three light bulbs and a single switch outside the room. 

The challenge is to determine which light bulb corresponds to which switch, using only one entry into the room.

Consulting firms may also present situational challenges as brain teasers.

For example, the "Airline Seating Problem" involves an airline flight with 100 seats and 100 passengers. The first passenger has lost their boarding pass and decides to sit in a random seat. Each subsequent passenger will either sit in their assigned seat or take a random unoccupied seat. 

The candidate must determine the probability that the last passenger will sit in their assigned seat.

Types of brain teasers you might face in consulting interviews  

problem solving interview riddles

There are seven common types of brain teasers that consulting firms may use in interviews.

Illusion question : 

Generating false impressions and focusing your attention on unimportant details and can lead you to miss the crucial information

Question 1:

Is it possible for a man in California to marry his widow's sister?

Question 2:

A farmer has 17 sheep and all but 9 die. How many are left?

Question 3:

How many two-cent stamps are there in a dozen?

Question 4:

If a doctor gives you three pills, telling you to take one every half hour, how many minutes will pass from taking the first pill to the last pill?

Question 5:

Two U.S. coins add up to 30 cents. If one of them is not a nickel, what are the two coins?

No. The word “his widow” signifies that the man has died.

9 sheeps. The question tries to lure you into calculating “17-9=8” when the answer is right there.

12 stamps. You didn't try to multiply 12 with 2, did you?

60 minutes. There are only 2 30-minute intervals, not 3.

A nickel (5 cents), and a quarter (25 cents). This question tricks you into thinking neither coin is a nickel.

Draw-explanation question : 

Providing you with a peculiar and improbable situation, and you have to develop an answer that fully explains the circumstances

A doctor's son's father was not a doctor. How is this possible?

A woman and daughter walked into a restaurant. A man walked past and the women both said “Hello, Father”. How is this possible?

Donald brought his wife to the hospital because she was suffering from appendicitis. The doctors removed her appendix. Five years later, the very same Donald brings his wife in, again for appendicitis. How is this possible?

A horse jumps over a castle, then lands on a man. The man disappears. What's happening?

A man was born in 1945, but he's only 30 years old now. How is this possible?

The doctor is the mother.

The man is a priest, or his name is Father.

Donald divorced and remarried.

1945 was the number of the hospital room.

Wording question : 

Including at least one word that can be interpreted into multiple meanings as well as misleadingly suggested by the context, usually different from the default interpretation in your mind, makes the question inexplicable 

What two words, when combined, hold the most letters?

People who smoke are much more likely to develop lung cancer than those who do not smoke. What research would possibly show that cigarette smoking does not cause cancer?

Make one word from all the following jumbled letters: R E O D N O W

What kind of cheese is made backwards?

What has four legs but only one foot?

“Post” and “office”.

One that shows an indirect relationship between smoking and lung cancer, i.e “smoking causes X, X causes lung cancer”. The key here is to look at “cause” as a direct relationship.

“One word”.

Edam cheese.

Pattern/trend questions : 

Involve a series of numbers or letters with a certain pattern or trend and your task is to either identify the following thing or fill in the gap.

What is the next number in the following sequence: 0 0 1 2 2 4 3 6 4 8 5 ?

What is the next letter in the following series: Y Z V W S T P Q ?

Which verb does not belong with the others in this set?

BRING BUY CATCH DRAW FIGHT SEEK TEACH THINK

MUSIC : VIOLIN is similar to:

(a) notes : composer / (b) sound : musical instrument / (c) crayon : drawing / (d) furniture : carpentry tools / (e) symphony : piano

What is the next number in the following sequence: 125, 64, 27, 8?

Number “10”

The sequence alternates between two different patterns:

Pattern 1: Incrementing by 1 (0, 1, 2, 3, 4, 5) 

Pattern 2:  Incrementing by 2 (0, 2, 4, 6, 8)

Letter “R”

The series alternates between two different patterns:

Pattern 1: Moving two letters forward in the alphabet (Y, V, S, P ...) Pattern 2: Moving one letter forward in the alphabet (Z, W, T, Q ...)

“THINK” is a mental process rather than a physical action

(b) sound : musical instrument

The next number in the sequence is 1=3^1.

125 = 5^3 

64 = 4^3 

27 = 3^3 

Logical questions : 

The least mind-bending format of them all, these questions typically don't include any creativity, illusions, or tricks, so you can solve them with just your pure math skills and logical reasoning.

A boy and a girl are sitting on a bench. “I’m a girl,” says the child with brown hair. “I’m a boy,” says the child with blond hair. If at least one of them is lying, which one is lying?

An explorer found a silver coin marked 7 BC. He was told it was a forgery. Why?

A bus can hold x people. It was half full from the start, and at the first stop, y people got off. How many people can now get on the bus?

What day follows the day before yesterday if two days from now will be Sunday?

During lunch hour a group of boys from Mr. Bryant’s homeroom visited a nearby grocery store. One of the five took an apple.

Jim said, “It was Hank or Tom”.

Hank said, “Neither Eddie nor I did it.”

Tom said, “Both of you are lying.”

Don said, “No, one of them is lying, the other is speaking the truth.”

Eddie said, “No, Don, that is not true.”

When Mr. Bryant was consulted, he said, “Three of these boys are always truthful but two will lie every time.”

Who took the apple?

Both are lying. If any of them told the truth, they would deliver one same answer.

At the start, the number of vacant seats on the bus was x/2. After the first bus stop it becomes x/2 + y.

Thursday. Today is Friday because “two days from now will be Sunday”. “day follows the day before yesterday” is just yesterday, so it’s Thursday.

Tom took the apple

Jim Hank, and Eddie were telling the truth

Tom and Don were lying

Letter-trick question:

These questions are twisted which plays with the organization demonstration and composition of letters, forcing us to consider them from a variety of perspectives.

What does this mean? “GGES EGSG SEGG ESGG”

What does this mean? “ROFORKAD”

What does this mean? “CCCCCCC”

What does this mean? “F AST”

What does this mean? “GR 12” AVE”?

Scrambled eggs.

Fork in the road.

Seven seas.

One foot in the grave.

Market-sizing and guesstimate questions : 

Guesswork questions that require respondents to make predictions about a topic in a given area using data to make informed decisions, and  effectively convey their thought process. 

This is one of the most popular types of brain teaser questions and is still widely used by recruiters. To gain a better understanding of this particular queries, you might read the article: Market-sizing & estimate questions.

→ You can also learn more about these 7 types of questions and how to solve them in the video: Solving ANY Brain Teasers After This Video. For Real!

Why Consulting Firms rarely use Brain Teasers in Interviews now?

Unfortunately, a lot of consulting firms have shifted away from using brain teasers in their interview processes for a number of reasons. Firstly, brain teasers may test a candidate's analytical and problem-solving abilities, they do not necessarily reflect the types of problems that consultants face in their day-to-day work .

Secondly, brain teasers can be intimidating for candidates and may not provide an accurate representation of their abilities . Candidates who are perfectly capable of solving complex business problems may struggle with brain teasers simply due to the pressure of the interview environment. This can result in an inaccurate assessment of a candidate's abilities and potential, leading to missed opportunities for both the candidate and the consulting firm.

Lastly, consulting firms have recognized the importance of diversity and inclusivity in their recruitment processes. Brain teasers may disadvantage candidates who have not had the opportunity to develop specific skills or who come from non-traditional backgrounds. 

By focusing on case interviews, which present candidates with real-world business problems that they may encounter as a consultant, consulting firms can provide a more level playing field for all candidates, regardless of their background or previous experience. This type of assessment is more reflective of the skills and abilities required for success as a consultant.

How to succeed in Consulting Interviews' Brain Teasers section?

Five ways to practice for brain teasers in consulting interviews.

Preparing for brain teasers in consulting interviews can be challenging, as they require a unique set of skills and abilities. However, with the right approach and practice, candidates can improve their performance and increase their chances of success. Below are some ways to practice for brain teasers in consulting interviews:

  • Start with basic brain teasers: Before tackling more complex brain teasers, it's essential to start with basic puzzles and problems. This will help you develop your problem-solving skills and build confidence in your abilities.
  • Focus on different types of brain teasers : There are many various types of brainteasers, such as word-based challenges, logical inquiries, and mathematics puzzles. Working on a number of different brainteasers to prepare yourself for the types of problems you may encounter in a consulting interview.
  • Practice online resources: There are many online resources available that offer practice brain teasers and puzzles. Websites like  BrainDen and Mentalup offer a variety of riddles and mental challenges, practicing in conjunction with Case Interview Programs will help you to be more equipped to handle a great situation during the interview.
  • Time management: In a consulting interview, time is often a factor, and candidates are expected to solve problems quickly and efficiently. Practice timing yourself when solving brain teasers to improve your speed and accuracy.
  • Practice under pressure: Consulting interviews can be high-pressure situations, and candidates may feel anxious or stressed during the interview process. Practice solving brain teasers under pressure to simulate the interview environment and prepare yourself for the real thing.

Five tips for solving brain teasers in consulting interviews

Similar to other tests and examinations, the brain teaser in consulting interviews also has some helpful tips to use. Some resharpers that may improve candidates' performance include:

  • Read the problem carefully:  The key to solving brain teasers is to understand the problem fully. Read the problem carefully, and make sure you understand what is being asked before attempting to solve the problem.
  • Break the problem down: Brain teasers can be complex and challenging, but breaking the problem down into smaller parts can help make it more manageable. Identify the key elements of the problem and work on solving each element individually before putting them together.
  • Use logic and reasoning: Try to use reasoning and creativity as you go through the brainteaser to come up with solutions. This may involve attempting different approaches, speculating, or using your understanding of science, math, or other disciplines to assist in solving the issue.
  • Draw diagrams and visualize the problem: Drawing diagrams and visualizing the problem can help you understand the problem better and find a solution. Use it in a way that helps you see the problem from different angles and perspectives.
  • Think outside the box: Brain teasers are designed to be challenging and require creative thinking to solve. Don't be afraid to think outside the box and explore unconventional solutions to the problem.

Four common mistakes when solving brain teasers during interviews

Nevertheless, there are some common mistakes that inexperienced candidates often have, preventing them from performing to their full potential. Here are a few things to steer clear of during the interview:

  • Making assumptions : Brain teasers often require candidates to make assumptions, but it's essential to make only necessary assumptions and not make assumptions that are not supported by the problem.
  • Explaining your thought unclearly: In a consulting interview, showing your thought is just as important as getting the right answer. Show your work and explain your thought process to demonstrate your problem-solving skills.
  • Rushing to a solution: Time is often a factor in consulting interviews, but rushing to a solution without fully understanding the problem can lead to mistakes and errors.
  • Overcomplicating the problem: Don't overcomplicate the problem by adding unnecessary complexity. Stick to the basics and use common sense to arrive at a solution.

Scoring in the McKinsey PSG/Digital Assessment

The scoring mechanism in the McKinsey Digital Assessment

Related product

Thumbnail of Case Interview End-to-End Secrets Program

Case Interview End-to-End Secrets Program

Elevate your case interview skills with a well-rounded preparation package

A case interview is where candidates is asked to solve a business problem. They are used by consulting firms to evaluate problem-solving skill & soft skills

There are 9 type of questions that mostly used in actual case interviews. Each type has a different solution, but you can rely on the a 4-step guide to answer

There are some questions or rather some principles about what candidates should and should not ask in an case interview to gain interviewer's approval

Career Sidekick

Interview Questions

Comprehensive Interview Guide: 60+ Professions Explored in Detail

8 Tough Brain Teaser Interview Questions from Google, Apple and Facebook

By Biron Clark

Published: November 16, 2023

Silicon Valley tech companies are famous for asking some pretty crazy brain-teaser interview questions… I wanted to find out exactly what these questions involve. And how difficult they are to answer. So I spent a day on Glassdoor.com and a few other sites to come up with the 8 hardest and most interesting logical interview questions out there. And not just from any companies…We’re going to look at 4 tech giants known for having the toughest interviews:

Time to see how many you can answer!

Facebook Brain Teaser Interview Questions and Answers:

Question 1:.

A Russian gangster kidnaps you. He puts two bullets in consecutive order in an empty six-round revolver, spins it, points it at your head and shoots. *click* You’re still alive. He then asks you, “do you want me to spin it again and fire or pull the trigger again right away?” For each option, what is the probability that you’ll be shot?

Answer…

The key hint here is that the bullets were loaded adjacent to each other.

There are 4 ways to arrange the revolver with consecutive bullets so that the first shot is blank. These are the possible scenarios:

The other two scenarios would have meant you got shot on the first attempt. (BBxxxx) or (BxxxxB)

Now look at the second slot in those 4 possible scenarios above. Your odds of getting shot are 1/4 or 25%. (Only #1 would get you shot)

But if you respin… there are 2 bullets remaining and 6 total slots. 2/6 or 33%.

Question 2:

You’re about to get on a plane to Seattle. You want to know  if it’s raining. You call 3 random friends who live there and ask each if it’s raining. Each friend has a 2/3 chance of telling you the truth and a 1/3 chance of messing with you by lying. All 3 friends tell you that “Yes” it is raining. What is the probability that it’s actually raining in Seattle?  

You only need 1 of your friends to be telling the truth for it to be raining in Seattle.

It’s fastest just to calculate the odds that all 3 are lying, and it’s not raining.

Each friend has a 1/3 chance of lying. Multiply the odds together… you get 1/27 (1/3 * 1/3 * 1/3).

We’re not done yet though… 1/27 is the probability that all 3 friends lied at the same time.

The probability that at least 1 told you the truth? 26/27 or around a 96% that it’s raining in Seattle.

Google Brain Teaser Interview Questions and Answers:

Question 3:.

You have a 3 gallon jug and 5 gallon jug, how do you measure out exactly  4 gallons?  

We know we can’t get the final result in the 3 gallon jug. It’ll overflow. We need to end up with 4 gallons in the 5 gallon jug.

First fill the 3 gallon jug.

Then pour the 3 gallons into the 5 gallon jug.

Now the 3 gallon jug is empty, and the 5 gallon jug has 3 gallons in it.

Fill the 3 gallon jug again. Slowly pour into the 5 gallon jug. Only 2 gallons will fit because it already has 3. Now it’s full.

Exactly 1 gallon is left in the 3 gallon jug.

Dump out the 5 gallon jug.

Pour your 1 gallon into the 5 gallon jug.

Fill up the 3 gallon jug one more time and pour it into the 5 gallon jug! You have exactly 4 gallons (and possibly a job at Google)

Question 4:

Why are manhole covers round?

Good news: If you’re tired of math questions this one will give you a break. Manhole covers are round because it’s the only shape that cannot fall through itself. The cover can never accidentally fall down the hole. Microsoft has been known to ask this question and according to Glassdoor.com, Google is asking this too now.

Apple Brain Teaser Interview Questions and Answers:

Question 5:  .

There are three boxes, one contains only apples, one contains only oranges, and one contains both apples and oranges. The boxes have been incorrectly labeled such that no label identifies the actual contents of its box. Opening just one box, and without looking in the box, you take out one piece of fruit. By looking at the fruit, how can you immediately label all of the boxes correctly?  

So, you know all 3 boxes are incorrectly labeled.

Go to the box labeled “Apples + Oranges.” Since the label is wrong, it must have one or the other.

This is the box to take one piece of fruit from. Whichever comes out is what that box contains. If you took out an apple, the box has only apples. If you took out an orange, vice versa.

Here’s where it gets tricky a bit tricky. But we’re almost done…

Let’s say you grabbed an apple. Move the “Apples” label over to that box. Now it’s correctly labeled.

You know the “Oranges” box is still labeled wrong (because all 3 were labeled wrong to start and you haven’t touched it). And you know it’s not “Apples”.

So it has to be “Apples + Oranges”.

The last box is “Oranges”.

The same process above would work if you had pulled out an orange at the start.

Question 6: 

You have 100 coins laying flat on a table, each with a head side and a tail side. 10 of them are heads up, 90 are tails up. You can’t feel, see or in any other way find out which 10 are heads up. Your goal: split the coins into two piles so there are the same number of heads-up coins in each pile.  

By pure coincidence… this is a trick my friend Mike showed me last summer. It blew my mind back then but hopefully it’ll make sense as I write it out.

You want an equal number of heads in each pile. There are currently 10 of them. You don’t know which but it doesn’t matter. All you have to do… take any 10 coins out of the 100, put them into a separate pile, and flip those 10 over.

That’s pile #1.

Pile #2 is the remaining 90 coins, unflipped. Just leave them.

You’re done. Seriously.

You can do this with any number of coins. If you had 20 coins, and 18 were heads, you’d need to take 18 of them (it doesn’t matter which) into a separate pile and flip those 18. That’s pile #1.

If you had 10 coins and 3 were heads, you’d take 3 random coins into a new pile and flip those 3 for your first pile, and the rest are your second pile.

Crazy right?

If you don’t believe me just grab some pennies and try it. There are no exceptions and it doesn’t need to be an even amount of “heads” to begin with either. It can also be zero. Or all.

LinkedIn Brain Teaser Interview Questions and Answers:

Question 7: .

You’re in a room with three light switches, each of which c ontrols one of three light bulbs in the next room. You need to determine which switch controls which bulb. All lights are off to begin, and you can’t see into one room from the other. You can inspect the other room only once. How can you find out which switches are connected to which bulbs?  

Let’s call the switches 1, 2, and 3.

Leave switch 1 off.

Turn switch 2 on for ten minutes.

Now turn it off and quickly turn on switch 3.

Go into the room and inspect…

The bulb that is still warm but not lit up is controlled by switch 2. The one that’s currently lit up is switch 3. The last one is switch 1.

Question 8: 

How many golf balls would fit into a Boeing 747?

This last one is tough, but they don’t expect you to get an accurate answer. If you get a question like this (and there are a ton of variations- basketballs in a room, cellphones in Manhattan, etc.) they want to see your thought process. The hiring manager is going to look at how you work your way through it and attempt to figure it out.

If you can break a problem down into smaller pieces, stay calm, and get an answer that’s not perfect but reasonably close, you’ve done great.

They might not even know the answer. They just want to see how you approach something that’s very difficult.

On a Practical Note, What Can You Take Away From This?

Question 8 above highlights a pretty good point to remember in your interviews…

There are a lot of questions in an interview where the hiring manager values your thought process… sometimes even more than a correct answer. So if you’re stumped, talk out loud a bit and explain what you’re thinking. Ask a question if you need to. Try to break it down into smaller pieces. Specific knowledge can be taught but they can’t teach you problem-solving. That’s why they ask logical questions in a job interview, and why they ask questions where they  expect you to struggle or be unsure.

If you hear questions like this, it doesn’t mean you’re doing badly. Just stay calm, walk them through your thought process, show you take a logical approach, and you’ll have a great shot at getting hired (even if you don’t come up with the perfect answer in the end!)

You Can Get Hired Even if You Give “Wrong” Answers to These Questions

Here’s a quick story: My degree is in Finance, which means I took a good amount of Accounting classes too. Early in my career, I had a phone interview for an Accounting position. To make the story short, I could not answer even the most basic accounting questions. Really simple stuff that you learn your first year in college.

It had just been too long since college and I had forgotten even the basics. And I didn’t prepare well for the interview obviously! But I tried to stumble through it and remember what I could, talking about what I was thinking. Saying things like, “well, this can’t be right because ___. So it must be related to ___.” I made some progress. But I definitely didn’t arrive at the right answer, even after three minutes of walking myself and the interviewer through it out loud.

But I still got invited to the next round in the interview process (a full day, on-site interview).

Why? Because the hiring manager liked my approach to breaking down a problem that I didn’t immediately know how to solve. That’s why being transparent and showing your thought process is one of the tips for interviews that you’ll see me say over and over. And that’s the biggest takeaway that I hope you gain from reading these brain teaser questions above (along with entertainment). You can do the same thing I did and get more job offers… even if you give a few wrong answers to difficult questions like these!

The bottom line is: Don’t panic when you get a question you don’t know; use it as an opportunity to show exactly how you work through things. Be confident with it, relax, and smile. Remember… you’re giving the hiring manager what they want! If you have interviews coming up and want to prepare further, read the top 20 interview questions here.

Like this post? Pin it to save for next time you need it!

8 toughest brain teaser job interview questions from google apple facebook

About the Author

Read more articles by Biron Clark

Continue Reading

12 Expert-Approved Responses to ‘What Makes You Unique?’ in Job Interviews

15 most common pharmacist interview questions and answers, 15 most common paralegal interview questions and answers, top 30+ funny interview questions and answers, 60 hardest interview questions and answers, 100+ best ice breaker questions to ask candidates, top 20 situational interview questions (& sample answers), 15 most common physical therapist interview questions and answers, 13 thoughts on “8 tough brain teaser interview questions from google, apple and facebook”.

I believe #1 is incorrect. I think it’s 1/5… am I missing something???

Question 3 can be done much more efficiently. All you have to do is to fill both jugs halfway. 3/2=1.5, 5/2=2.5, 1.5+2.5=4. It’s easier to estimate a half filled jug than filling a 3 gallon jug by one third. The only downside to this solution is that uncertainty is higher.

This is old: the research found brain teaser questions don’t do anything to evaluate job candidates other than stressing out the interviewee. Google has stopped using brain teasers.

Really interesting and good questions

Apologies if this has already been mentioned, but your answer to Q2 is not correct. You should be trying to solve the conditional probability problem P(Rain | YYY), and NOT 1 – P(lie, lie, lie).

The tricky part here is that you are not given all of the information you need to apply Bayes rule and solve the problem. Specifically, you additionally need to have a prior estimate of P(Rain). The interviewer/question purposefully withholds this information to see if you are able to identify how the problem should be solved, and if you can ask the right clarifying questions.

The solution, using Bayes Rule, is;

P(Rain | YYY) = P(Rain) * P(YYY | Rain) / P(YYY)

P(Rain) must come from the interviewer P(YYY) = P(Rain) * P(YYY | Rain) + P(~Rain) * P(YYY | ~Rain) P(YYY | Rain) = (2/3)^3 # All 3 friends tell the truth when it rains P(YYY | ~Rain) = (1/3)^3 # All 3 friends lie when it’s not raining

I hope this helps. Check out the following link for an excellent explanation and some further discussion of Bayesian vs frequentist approaches:

#3 is a correct solution but more complex than it needs to be. These companies also look for efficiency and not doubling your efforts.

Your Solution: 1. Fill up 3gal Bucket > Dump into 5gal bucket 2. Fill up 3gal Bucket > Dump 2gal into the remaining 2gal of area within the 5gal bucket 3. Empty 5gal Bucket > Dump 1gal within the 3gal bucket into the empty 5gal bucket 4. Fill up 3gal Bucket > Dump 3gal bucket completely into 5gal bucket that currently has 1gal within it.

This solution requires you to empty a bucket down the drain 1 time, fill a bucket from the faucet 3 times, and pour a bucket into a bucket 4 times.

Efficient Answer

1. Fill up 5gal bucket completely > Pour 3gal from 5gal bucket into the 3gal bucket to fill it up completely 2. Empty full 3gal bucket > Pour remaining 2gal from the 5gal bucket into the empty 3gal bucket 3. Fill up the 5gal bucket completely > Pour 1gal from the full 5gal bucket into the remaining 1gal of volume within the 3gal bucket.

This solution requires you to empty a bucket down the drain 1 time, fill a bucket from the faucet 2 times, and pour a bucket into a bucket 3 times.

If they ask you to solve the same problem, but dumping as little as possible down the drain, solution 2 is still your best bet. (1=5gal dumped, 2=3gal dumped because the problem is solved at the precise time you would need to dump it again for another step)

The only way that solution 1 is more efficient is if they ask you to solve the same problem USING as little water as possible. (1=9gal, 2=10gal)

Interesting. Thanks for the detailed reply.

The solution to Question 2 is incorrect.

You argue that the probability that at least one friend tells the truth is one minus the probability that all three friends are lying. This is correct. But you then say that we only need one friend to tell the truth for it to actually be raining. This may be true, but it takes into account scenarios where one friend is lying and the the other two are telling the truth, and vice versa. However, this is ignoring a key element: we already know that this has not happened! If all three friends are claiming that it is raining, then it is impossible that one is lying and two are telling the truth (analogously, it is impossible that two are lying and one is telling the truth) because they all said the same thing (and they know whether or not it is raining!).

One of two situations is possible: either it is raining in Seattle, or it is not. If (and only if) it is raining, then they are all telling the truth. If (and only if) it is not, then they are all lying. The probability that they are all telling the truth is 8/27; the probability that they are all lying is 1/27. The probability that they are telling the truth given that they all said the same thing is (8/27)/(1/27 + 8/27) = 8/9.

You are right. I also came to the answer 8/9 and wanted to comment that the given solution is wrong. I came across your comment and decided to just reply here :)

You’re right that the solution is incorrect, and you’re on the right track, but there’s actually not enough information, as the answer is dependent on the prior probability of it raining. If you let this prior probability be p, and apply Bayes’ rule, you should get that the probability that it’s actually raining, given that all three friends said yes, is 8p/(7p+1). If (and only if) you let p=1/2, then you recover an answer of 8/9.

This blog was… how do I say it? Relevant!! Finally I have found something which helped me. Cheers!

Are they asking questions to everyone or just software developers?? It seems geared toward developers mostly

It’s everyone, although you’ll expert more of these if your position involves analytical thinking, logic, problem solving, etc. Many Product Managers will face these questions. Also corporate lawyers! And many others along with software developers.

Comments are closed.

Resumehead

  • Career Blog

8 Tough Brain Teaser Interview Questions and Answers in 2024

problem solving interview riddles

Brain teaser interview questions, also known as puzzle or logic questions, are designed to challenge candidates and evaluate their problem-solving, critical thinking, and creativity skills. They are usually asked in job interviews for positions that require analytical thinking or problem-solving abilities. Companies use these questions to assess a candidate’s ability to respond to unexpected and challenging situations, which can indicate their potential to excel in the role.

The importance of brain teaser questions lies in their ability to go beyond a candidate’s resume or credentials. Unlike traditional interview questions, which may have a predetermined answer or are related to a candidate’s job experience, brain teasers can reveal how a candidate thinks and approaches complex problems. They can also demonstrate a candidate’s ability to think quickly, be creative, and demonstrate their character and personality under pressure.

Common Types of Brain Teaser Interview Questions

Brain teasers can take many different forms, but they all have one common feature: they are designed to challenge you to think creatively and logically. Some of the common types of brain teaser interview questions include:

  • Logic puzzles: These questions involve various scenarios or situations designed to test your logical reasoning skills.
  • Numerical puzzles: These questions involve mathematical problems or puzzles that require you to use your math skills to solve.
  • Lateral thinking puzzles: These are puzzles that can be solved through creative or unorthodox thinking.
  • Competency-based questions: These questions are often hypothetical and ask you to demonstrate how you would respond in a specific situation.

The Purpose of Brain Teaser Interview Questions

The purpose of brain teaser questions is to assess how a candidate thinks and approaches complex problems. They are often used by companies to evaluate a candidate’s potential rather than their specific skills or experience. Brain teaser questions may also be used to evaluate a candidate’s ability to work under pressure, think creatively, be adaptable, and tackle complex problems. By asking these questions, employers can gain insights into how candidates solve problems and make decisions, which can be valuable in assessing their potential for the job.

Brain teaser interview questions are designed to challenge candidates and evaluate their problem-solving, critical thinking, and creativity skills. They are an important tool for hiring managers to assess the potential of candidates and provide insights into how they approach complex problems. It is essential for candidates to prepare for these questions to demonstrate their potential to excel in the role.

How to Prepare for Brain Teaser Interview Questions

When preparing for a brain teaser interview, it’s important to keep in mind that these types of questions are designed to test your problem-solving and critical thinking skills. With a few tips and best practices, you can feel more confident and prepared to tackle these challenging interview questions.

A. Tips for Answering Brain Teaser Interview Questions

Take your time: Brain teaser questions are meant to be challenging and require careful thought. Take a moment to gather your thoughts and ask clarifying questions before jumping into an answer.

Break down the problem: Often, brain teasers are presented as complex problems that can seem overwhelming. Break the problem down into smaller parts to identify patterns and possible solutions.

Use logic and reasoning: Brain teasers often involve patterns, sequences, or logic puzzles. Use your analytical skills to identify commonalities and solve the problem logically.

Show your work: As you work through a brain teaser, be sure to share your thought process with the interviewer. This can demonstrate your problem-solving skills and give insight into how you approach difficult situations.

B. Best Practices for Preparing for Brain Teaser Interview Questions

Research common brain teasers: There are many brain teasers that are commonly asked during interviews. Familiarize yourself with these questions and practice solving them ahead of time.

Practice under pressure: In addition to researching and solving brain teasers, try practicing with time constraints. This can simulate the pressure of an interview and prepare you for thinking on your feet.

Challenge yourself: As you prepare for brain teaser questions, don’t shy away from difficult problems. Push yourself to solve more complex challenges to build your problem-solving skills.

Get feedback: Practice solving brain teasers with a friend or colleague and ask for feedback on your performance. This can help you identify areas for improvement and refine your approach.

C. The Benefits of Preparing for Brain Teaser Interview Questions

Preparing for brain teaser interview questions can have numerous benefits for your career. Not only does it demonstrate your analytical and problem-solving skills, but it can also improve your overall critical thinking abilities. Additionally, practicing under pressure can help you feel more comfortable in high-stress situations, which can be beneficial in any work environment. By taking the time to prepare for brain teaser questions, you can feel more confident and prepared to tackle any challenge that comes your way during an interview.

The 8 Tough Brain Teaser Interview Questions and Answers

As a job seeker, it’s important to be prepared for any questions that may come your way during an interview, including tough brain teasers. These questions are designed to assess your problem-solving abilities and critical thinking skills. Here are eight examples of tough brain teaser interview questions and answers that you can use to help you prepare for your next job interview:

Question 1: Example Question and Answer

Q: How many golf balls can fit in a school bus?

A: To answer this question, you need to make some assumptions. Let’s assume the school bus is 40 feet long, 8 feet wide, and 6 feet high. That means it has a volume of 1,920 cubic feet. Now, let’s assume a golf ball is 1.68 inches in diameter. That means each golf ball has a volume of about 2.65 cubic inches. To fill the entire school bus with golf balls, we need to convert the volume of the school bus to cubic inches and then divide by the volume of a single golf ball.

  • Convert school bus volume to cubic inches:

40 ft x 12 in/ft = 480 in (length) 8 ft x 12 in/ft = 96 in (width) 6 ft x 12 in/ft = 72 in (height) 480 in x 96 in x 72 in = 331,776 cubic inches

  • Divide school bus volume by golf ball volume:

331,776 cubic inches ÷ 2.65 cubic inches per golf ball = 125,233 golf balls

So, the answer is approximately 125,233 golf balls can fit in a school bus.

Question 2: Example Question and Answer

Q: How would you design a spice rack for blind people?

A: This question is designed to assess your creativity and problem-solving abilities. A possible answer might include:

  • Use different textures or shapes on each spice bottle to allow for easy differentiation by touch.
  • Label the spices using braille or embossed letters.
  • Make the spice rack adjustable or modular to accommodate different types and sizes of spice bottles.
  • Include a simple guide or key to help users easily locate the desired spice.

Question 3: Example Question and Answer

Q: You are standing in front of three light switches. One of them controls a light bulb inside a closed room. You cannot see into the room. You can only open the door once, and after that, you cannot touch the switches. How do you figure out which switch controls the light bulb?

A: To solve this brain teaser, you need to follow a specific set of steps:

  • Turn on one of the switches and leave it on for a few minutes.
  • Turn off that switch and turn on another switch.
  • If the light bulb is on, then you know that the switch you just turned on controls the light bulb. If the light bulb is off but warm to the touch, then you know that the switch you initially turned on and then turned off controls the light bulb. Finally, if the light bulb is off and cool to the touch, then the remaining switch controls the light bulb.

Question 4: Example Question and Answer

Q: You have a 3-gallon jug and a 5-gallon jug. How can you measure exactly 4 gallons of water using only these two jugs?

A: Here’s a step-by-step solution:

Fill the 5-gallon jug to its maximum capacity. Pour the 5 gallons of water from the 5-gallon jug into the 3-gallon jug, which will leave 2 gallons of water in the 5-gallon jug. Empty the 3-gallon jug. Pour the 2 gallons of water from the 5-gallon jug into the empty 3-gallon jug. Fill the 5-gallon jug to its maximum capacity again. Pour enough water from the 5-gallon jug into the 3-gallon jug until the 3-gallon jug is full. This will leave 4 gallons of water in the 5-gallon jug.

Now you have exactly 4 gallons of water in the 5-gallon jug.

Question 5: Example Question and Answer

Q: How many trailing zeros are there in the number 100 factorial (100!)?

A: To determine the number of trailing zeros in 100!, you need to identify the number of factors of 5 since multiples of 10 (which have one trailing zero) are created by the combination of factors of 2 and 5.

Counting the number of multiples of 5 less than or equal to 100 gives you 20. However, there are multiples of 5 squared (25), multiples of 5 cubed (125), and so on. So, you need to consider the additional multiples of 5^2, 5^3, and so on that divide 100.

Dividing 100 by 5^2 gives you 4, and there are no multiples of 5^3 or higher within the range of 1 to 100. Therefore, the total number of trailing zeros in 100! is 20 + 4 = 24.

Question 6: Example Question and Answer

Q: How many squares are there on a chessboard?

A: To count the number of squares on a chessboard, you need to consider squares of different sizes. Start by counting the 64 individual squares (1×1). Then, count the 49 squares that are formed by combining four individual squares (2×2). Continue this process for squares of size 3×3, 4×4, and so on until you reach the largest square, which is the entire chessboard (8×8).

The total number of squares on a chessboard is the sum of all these counts:

1×1 squares: 64 2×2 squares: 49 3×3 squares: 36 4×4 squares: 25 5×5 squares: 16 6×6 squares: 9 7×7 squares: 4 8×8 squares: 1

Adding these counts together, you get a total of 204 squares on a chessboard.

Behavioral Questions vs. Brain Teaser Questions

A. definition and differences between behavioral and brain teaser questions.

During job interviews, employers often ask job seekers a variety of questions to assess their problem-solving, critical thinking, and communication skills. Two types of questions that you may encounter during a job interview are behavioral and brain teaser questions.

Behavioral Questions:  Behavioral questions inquire about how you have handled previous work situations or challenges. These questions are designed to allow the interviewer to understand your past work experiences, your response to various situations, and your attitude towards work. Behavioral questions require you to provide specific examples of how you have handled specific work scenarios in the past.

Brain Teaser Questions:  Brain teaser questions are designed to assess your thought process, problem-solving capabilities, and creativity. These types of questions do not necessarily have a right or wrong answer. Typically, brain teaser questions are seemingly unrelated to the job or industry and may be puzzles, riddles, or other types of mind-bending challenges that rely on logic and critical thinking to solve.

The fundamental difference between behavioral and brain teaser questions is that behavioral questions look at how you have responded to situations in the past, while brain teaser questions assess your ability to think on your feet and solve unfamiliar problems quickly.

B. How to Prepare for Behavioral and Brain Teaser Questions

The best way to prepare for behavioral and brain teaser questions is to practice. Try to anticipate which types of questions may be asked and prepare specific examples to share during the interview.

For behavioral questions, review the job description and requirements before the interview. Then, think of specific examples that demonstrate how you have handled similar situations in the past. Make sure to use the STAR technique (Situation, Task, Action, Result) when answering behavioral questions to showcase your problem-solving skills effectively.

When it comes to brain teaser questions, the best way to prepare is by working on logic puzzles, brainteasers, and riddles. You can find plenty of resources online or in books to help you practice. Work on puzzles that stretch your mind and challenge you to expand your thinking.

C. Tips for Answering Behavioral and Brain Teaser Questions

Here are some tips to keep in mind when answering behavioral and brain teaser questions:

Behavioral Questions:

Be specific: Use concrete examples to illustrate your answer. Provide clear details of what you did and why you did it.

Focus on your actions: Highlight the actions you took and the decisions you made in response to the situation.

Use the STAR technique: The STAR technique is Situation, Task, Action, and Result. Use this framework to structure your answers and showcase your problem-solving abilities effectively.

Brain Teaser Questions:

Clarify the question: Ask the interviewer to repeat or explain the question if you don’t understand it fully.

Take your time: Don’t rush to answer the question.

The Role of Brain Teaser Questions in the Hiring Process

When it comes to the hiring process, many companies are turning to brain teaser questions as a way to assess a candidate’s critical thinking and problem-solving abilities. Here, we will explore how hiring managers use brain teaser questions, the benefits of using them, and the limitations that should be considered.

A. How Hiring Managers Use Brain Teaser Questions

Hiring managers use brain teaser questions to get a sense of a candidate’s ability to think on their feet, approach problems creatively, and work through challenging situations. These types of questions often require the candidate to use logic, reasoning, and unconventional thinking to come up with a solution.

Some hiring managers use brain teasers as a way to gauge a candidate’s personality and how they handle pressure. These questions can reveal a lot about how a candidate works under stress, their communication skills, and how they adapt to change.

B. The Benefits of Using Brain Teaser Questions

There are several benefits to using brain teaser questions in the hiring process. First and foremost, they allow hiring managers to quickly assess a candidate’s critical thinking skills, which are essential for success in many roles.

Brain teasers also help hiring managers get a sense of a candidate’s personality and how well they may fit into the company culture. By asking questions that require unconventional thinking and problem-solving, hiring managers can get a sense of how a candidate approaches challenges and whether they are a good fit for the team.

C. The Limitations of Brain Teaser Questions

While brain teaser questions can be a useful tool in the hiring process, it is important to recognize their limitations. First and foremost, they should not be the sole criteria on which a candidate is evaluated.

It is also important to recognize that brain teaser questions may not accurately reflect a candidate’s ability to perform the specific job they are applying for. While these questions can be useful in assessing critical thinking and problem-solving skills, they may not be relevant to the day-to-day tasks of the job.

Finally, it is important to ensure that brain teaser questions are fair and not biased against certain candidates. Questions should be designed to challenge all candidates equally, regardless of their background or experience.

Brain teaser questions can be a useful tool in the hiring process, allowing hiring managers to quickly assess a candidate’s critical thinking skills and personality. However, it is important to recognize their limitations and ensure they are used fairly and effectively.

The Impact of Brain Teaser Questions on Job Performance

Incorporating brain teaser questions during the hiring process has become increasingly popular as hiring managers aim to assess a candidate’s ability to think critically under pressure. However, there is a debate on whether brain teasers accurately predict job performance.

A. The Relationship between Brain Teaser Performance and Job Performance

Some studies suggest that there is a positive correlation between brain teaser performance and job performance. Candidates who score well on brain teasers tend to perform better on the job. This is because brain teasers test a candidate’s ability to think logically, creatively and make sound decisions when faced with complex problems.

B. How Hiring Managers Evaluate Brain Teaser Performance

Hiring managers evaluate brain teaser performance in different ways. Some use it as a primary factor in deciding whether to hire a candidate, while others use it as one of many factors to evaluate a candidate. They assess a candidate’s performance by examining how quickly and efficiently they answer the questions, their thought processes, and the accuracy of their answers.

C. The Potential Risks of Overemphasizing Brain Teaser Performance

While brain teasers can be an effective tool for evaluating candidates, they also come with potential risks. Overemphasizing brain teaser performance may lead to the exclusion of highly qualified candidates who may not perform well under pressure. It may also place undue stress on candidates, leading to anxiety and nervousness that can negatively impact their performance.

Furthermore, brain teasers may only assess an individual’s problem-solving ability, whereas other important job-specific skills may not be evaluated. Thus, it is essential to use brain teasers as part of a more comprehensive evaluation process.

While brain teasers can provide valuable insights into a candidate’s critical thinking ability, it should not be the sole criterion for evaluating job performance. It is important to use a combination of various assessment tools to ensure a fair and accurate evaluation of a candidate’s suitability for the role.

The Future of Brain Teaser Questions in the Hiring Process

A. trends in the use of brain teaser questions.

As the job market becomes increasingly competitive, hiring managers are looking for new ways to identify the top candidates for their open positions. One trend that has emerged in recent years is the use of brain teaser questions during the hiring process.

Brain teaser questions are designed to test a candidate’s problem-solving skills, creativity, and ability to think on their feet. They can be used in a variety of industries and job roles, from finance and accounting to marketing and technology.

Some companies use pre-designed brain teaser questions, while others prefer to create their own based on the specific job requirements. As the use of brain teaser questions becomes more widespread, it’s important for hiring managers to ensure that the questions are relevant to the job and provide a true measure of a candidate’s abilities.

B. The Benefits and Risks of Using Brain Teaser Questions

The use of brain teaser questions in the hiring process has both benefits and risks. On the one hand, they can help managers identify candidates who are able to think critically and creatively, which may be particularly important for roles that require problem-solving skills.

Brain teasers can also be a good way to differentiate between candidates who may have similar qualifications and experience. Additionally, they can give candidates a chance to showcase their skills in a way that traditional interview questions may not.

However, there are also risks associated with using brain teaser questions in the hiring process. For one, they may not be a fair indicator of a candidate’s overall abilities or potential for success in a role. Additionally, some candidates may find them intimidating, which could create a negative impression of the company.

To mitigate these risks, it’s important for hiring managers to use brain teaser questions in conjunction with other types of interview questions and assessments. Additionally, they should be transparent about the use of brain teasers and provide candidates with adequate preparation time before the interview.

C. Future Innovations in Brain Teaser Interviews

As technology continues to evolve and change the way we work, it’s likely that we’ll continue to see innovations in the use of brain teasers during the hiring process. One trend that may emerge is the use of virtual reality or augmented reality to provide candidates with more dynamic and engaging brain teaser experiences.

Another potential innovation is the use of machine learning algorithms to analyze a candidate’s responses to brain teaser questions and provide more nuanced insights into their problem-solving abilities.

Ultimately, the future of brain teaser questions in the hiring process will be shaped by a variety of factors, including advances in technology, changes in the job market, and evolving attitudes toward hiring practices. As hiring managers continue to explore the use of brain teasers, it will be important to strike a balance between their potential benefits and risks.

Related Articles

  • Store Associate Job Description: A Blueprint for Success
  • HR Business Partner Resume: Examples and Tips for 2023
  • How to Write a Great Resume in 2023: 101 Full Guide
  • The Ultimate Teacher Resume Template: Example for 2023
  • Technology Skills to Include in Resume and Cover Letter

Rate this article

0 / 5. Reviews: 0

More from ResumeHead

problem solving interview riddles

Explore Jobs

  • Jobs Near Me
  • Remote Jobs
  • Full Time Jobs
  • Part Time Jobs
  • Entry Level Jobs
  • Work From Home Jobs

Find Specific Jobs

  • $15 Per Hour Jobs
  • $20 Per Hour Jobs
  • Hiring Immediately Jobs
  • High School Jobs
  • H1b Visa Jobs

Explore Careers

  • Business And Financial
  • Architecture And Engineering
  • Computer And Mathematical

Explore Professions

  • What They Do
  • Certifications
  • Demographics

Best Companies

  • Health Care
  • Fortune 500

Explore Companies

  • CEO And Executies
  • Resume Builder
  • Career Advice
  • Explore Majors
  • Questions And Answers
  • Interview Questions

Great Brain Teaser Interview Questions (With Answers)

  • Situational Interview Questions
  • Promotion Interview Questions
  • Internal Interview Questions
  • Open Ended Interview Questions
  • Tough Interview Questions
  • Leadership Interview Questions
  • Teamwork Interview Questions
  • Interview Questions About Communication
  • Personality Interview Questions
  • Internship Interview Questions
  • Ice Breaker Questions
  • Recruiter Interview Questions
  • Brain Teaser Interview Questions
  • Group Interview Questions
  • Competency Based Interview Question
  • Grad School Interview Questions
  • Scrum Interview Questions
  • Excel Interview Questions
  • Common Phone Interview Questions And Answers

Find a Job You Really Want In

Interviewers will often ask brain teaser interview questions because they will help assess your workplace skills. Answering brain teasers correctly can involve critical thinking , problem-solving , creativity , and close listening skills . For these reasons, they’re commonly used during interviews to see how well and how quickly a candidate can think on his or her feet.

Whether you’re thinking about using brain teaser questions to test a candidate’s skills or you’re preparing for an interview yourself, this article will go over what brain teaser questions are, some examples of open and closed brain teaser questions, and the pros and cons of brain teasers for interviews.

Key Takeaways:

Brain teaser questions test your analytical and problem-solving skills in real time.

Some brainteaser interview questions only have one right answer, while others are more open-ended.

Listen closely when asked a brain teaser question and clarify all information.

Do your thinking out loud. It is just as important for the interview to see how you think as it is to get the right answer.

Brain teaser questions are not great for evaluating visual learners.

[Great Brain Teaser Interview Questions (With Answers)

What are brain teasers?

Close ended brain teaser question examples, open ended brain teaser question examples, why interviewers ask brain teaser questions, tips for answering brain teaser interview questions, brain teaser question pros and cons, brain teaser interview question faq.

  • Sign Up For More Advice and Jobs

Brain teasers are a form of puzzle that tests one’s thinking and problem-solving abilities. With brain teasers, the answer is never straightforward. Instead, brain teasers require a creative thought process known as lateral thinking, or arriving at answers in unconventional ways. They may also involve the creative use of mathematics and formal logic.

Brain teaser questions can reveal a lot about how a person thinks, which is why they are such a popular option for job interviews. Often, your answer ends up being less important than how you arrived at your answer – the hiring manager values being able to see your thought process.

There are many different kinds of brain teasers, all involving various types of intelligence and thinking to solve. Some might require math skills, others may test your close listening abilities, and others might rely on creativity and boldness.

A brain teaser might have one right answer, it may have multiple solutions, or it may be open-ended with no definite correct answer. Let’s take a look at some examples to give you a better idea.

Close ended brain teaser questions have a “right” answer that you arrive at through creative problem-solving. Here are some examples of this type of brain teaser question:

“You have a three-gallon bucket and a five-gallon bucket. How do you measure out exactly four gallons?”

Answer: The three-gallon bucket is too small, and the five-gallon bucket is too large. This is a given fact, and along with it comes the assumption that our final four gallons will end up in the five-gallon bucket. The first step is to fill the three-gallon bucket. Then, pour all three gallons into the five-gallon bucket. Now the five-gallon bucket has three gallons in it, and the three-gallon bucket is empty. Because 5-3=2, we know that only two more gallons can fit into the five-gallon bucket before it is full. So we’d fill the three-gallon bucket again and slowly pour it into the five-gallon bucket until it’s full. 3-2=1, so there is one gallon in the three-gallon bucket. We dump out the five-gallon bucket completely, then pour the one-gallon into it. Finally, we fill up the three-gallon bucket and pour it into the five-gallon bucket. We end up with precisely four gallons.

“The Beatles need to cross a bridge at night to get to a concert. They only have one flashlight, and they have seventeen minutes to get there. The bridge must be crossed with the flashlight and can only support two people at a time. John can cross in one minute, Paul can cross in two minutes, George can cross in five minutes, and Ringo takes ten minutes to cross. How can they all make it to the concert on time?”

Answer: First, John takes the flashlight and crosses the bridge with Paul. This takes two minutes. John then returns across the bridge with the flashlight taking one more minute (three minutes have passed so far). John gives the flashlight to George, and George and Ringo cross together, taking ten minutes (thirteen minutes have passed so far). George gives the flashlight to Paul, who recrosses the bridge taking two minutes (fifteen minutes have passed at this point). John and Paul now cross the bridge together, taking two more minutes. All end up across the bridge at the concert in exactly seventeen minutes.

“You are standing outside of a room that has no windows. The room has three light bulbs and three switches outside of the room. Each switch controls one of the light bulbs. You can only enter the room one time. How do you find out what switch goes to which light bulb?”

Answer: Turn on the first light switch and wait five minutes or so. After this time, turn off the first light switch and turn on the second switch. At this point, one light bulb (corresponding to the second switch) will be on, and one light bulb (corresponding to the first switch) will be turned off but warm from being on previously. Walk into the room, and feel the two turned-off light bulbs. The warm light bulb belongs to the first switch, the turned-on light bulb belongs to the second switch, and the remaining light bulb belongs to the third switch.

“There are three boxes, one box labeled “bananas,” another labeled “strawberries” and the last labeled “mixed.” All the boxes are labeled incorrectly. You’re only allowed to reach into a single box and take out one piece of fruit. Without looking into the box, how will you fix the labels?”

Answer: The first step is to open the box labeled “mixed” first. Since none of the boxes are labeled correctly, it’s guaranteed that this box doesn’t contain a mix of fruits. Say I picked a banana. This lets me know that the box labeled “mixed” is actually the “bananas” box. Now that the box labeled “mixed” is actually bananas, I can deduce that the box originally labeled “bananas” must be the “strawberries” box and the remaining box must be the “mixed” box. I would then label each box with their correct label.

Open ended brain teaser questions have no single right answer that you have to figure out. Instead, these questions can be answered in any way that makes sense to you and are meant to show off your unique approach to making sense of what’s asked of you.

“How many people are using Instagram in San Francisco at 1:30 pm on a Thursday?”

How to answer: This question not only tests your knowledge of certain crucial pieces of data (e.g., knowing how many people use Instagram at a given time throughout the day, knowing how many people live in San Francisco), but it also tests how you combine these different pieces of knowledge (or educated guesses). This question may also take the form of “How many [things] are there in [place]? To answer this and similar questions, utilize educated guesses about the various parts involved and develop a formula that makes sense to you.

“How would you describe a sunset to a blind person?”

How to answer: This question tests your knowledge and grasp of a given concept, as well as being an opportunity to display tremendous amounts of creativity in your answer. This question might also take the form of “Explain [a difficult concept] to [someone it would be difficult to explain it to].” Dig deeply into your powers of creative description, as well as everything you know about the concept or thing you are meant to describe. Remember that being able to teach a concept at varying levels is a mark of understanding it very well, so lean heavily on your knowledge of the subject.

“Sell me my iPhone.”

How to answer: This is a brain teaser as well as a practical test of your skills as a salesperson. The interviewer will pick an object around the room close by, such as their phone or a pen, and ask you to convince them to buy it. It may seem like a difficult, nerve-wracking task at first, but remember that they already own this item, so there are already a wealth of reasons for you to work with. The best strategy is to emphasize the need – why does this person need this thing? What value does it serve? However, rather than simply stating the facts, try and paint a vivid picture. Stimulate the interviewer’s imagination, perhaps, by describing in detail a time when this object may be a lifesaver.

“How much should you charge to wash all the windows in Seattle?”

How to answer: This answer can be answered in a different way but it provides you an opportunity to give a simple answer of one number. A popular response to this question would be that there is no need to wash the windows in Seattle because it rains so much that they’re washed naturally. Another way to answer this is to estimate the number of buildings and thus windows. How you answer this question will reveal how you approach thinking about abstract questions. It will test your quick thinking, your creative problem-solving, and your mathematical thinking.

Interviewers use brain teaser questions during interviews as an alternative way of getting to know a candidate and getting a deeper insight into how they might solve an unexpected problem. Brain teasers are asked alongside more typical interview questions and help form a more complete understanding of a candidate.

As mentioned earlier, asking brain teaser questions isn’t about seeing whether a potential employee has the right answers, but seeing how they think about and approach difficult problems.

Brain teasers can be used to reveal a candidate’s skills and abilities. They give crucial insight into how someone assesses and utilizes information and how they solve problems. They can also provide a good look into how well a potential employee deals with stress , how attentively they listen, and how well they can eloquently present their solutions.

When answering brain teaser questions, make sure you are prepared by reviewing some common brain teaser questions and then listening to the interviewer closely. Here are some more tips to keep in mind when answering brain teaser interview questions:

Come prepared. First things first, you’ll want to understand the underlying aims of asking these types of questions. Questions like these can provoke our inner test anxiety but understand that the interviewer is simply trying to get a picture of how you work through and communicate difficult issues.

Take a couple of deep breaths before your interview (and remember to stay breathing during the process) to help your nerves settle and bring about a state of mind that’s ready to take on challenging tasks. Bring a pen and paper to your interview to take notes or work out calculations.

Listen closely. One of the most important things you can do to accurately and cleverly solve a brain teaser is to listen attentively to the words. Many brain teasers rely on small but crucial pieces of information getting past you, so close listening is always the best first step in answering these questions.

Don’t try to answer the questions as they’re being asked to you or immediately after. With this strategy, you’re sure to become attached to certain pieces of information while ignoring other (potentially vital) pieces of information. Sometimes the answer can even be hidden within the question, so pay attention.

Do all your thinking out loud. Remember that brain teaser questions are asked to get insight into your thinking. State every thought you have in regard to answering this question out loud. Any assumptions or estimations that you make should be stated to the interviewer.

Make sure you have a methodical approach to answering the question. Come up with a working strategy that you can use and take the interviewer along for the ride. Come up with a formula for mathematical questions.

Brain teaser questions are very useful in a job interview. However, they are not perfect. The advantages and drawbacks of brain teaser questions reveal how hiring the right employee takes a lot of effort.

Pros of brain teaser questions include:

Shows how someone deals with pressure. Being put on put on the spot to solve a problem is a great way to analyze how someone thinks on their feet.

Reveals an ability to analyze information. One of the key components of a brain teaser question is taking the information that is given to solve a problem. This requires well-developed analytical skills.

Shows an ability to communicate solutions. Not only do you need to come up with a correct answer for a brain teaser, you need to communicate that answer effectively. A proper response to a brain teaser question shows that you can organize your thoughts.

Cons to brain teaser questions include:

The candidate might already know the answer ahead of time. Consider how this article gives you some example answers. If you are asked these questions in an interview, then the point of the question is unfulfilled.

Difficult for visual learners. Following verbal instructions (as opposed to written instructions) is more challenging for visual learners, and this may translate to an incorrect assessment of their skills.

The answer doesn’t reveal many skills. Although a correct answer to a brain teaser question reveals some skills, it does not reveal every skill needed in the workplace.

What if I get the brain teaser wrong in an interview?

Most of the time the answer to the brain teaser isn’t the main reason the interviewer is asking the question. Interviewers will ask brain teaser questions to gain insight into your use of logic and how you are able to answer questions under pressure. Brain teasers will also often test your analytical, active listening, and problem-solving skills, which are skills interviewers want to know about.

Why do interviewers ask brain teaser questions?

Interviewers will ask brain teaser questions to get an understanding of your critical thinking skills. It will also give them a deeper understanding of how you solve problems and who you are as a candidate. The questions are often asked not to get the correct answer but to see how the candidate comes up with their answer.

MConsulting Prep – 30 Brain Teaser Interview Questions & Detailed Answers

U.S. Department of Labor – Interview Tips

How useful was this post?

Click on a star to rate it!

Average rating / 5. Vote count:

No votes so far! Be the first to rate this post.

' src=

Chris Kolmar is a co-founder of Zippia and the editor-in-chief of the Zippia career advice blog. He has hired over 50 people in his career, been hired five times, and wants to help you land your next job. His research has been featured on the New York Times, Thrillist, VOX, The Atlantic, and a host of local news. More recently, he's been quoted on USA Today, BusinessInsider, and CNBC.

Recent Job Searches

  • Registered Nurse Jobs Resume Location
  • Truck Driver Jobs Resume Location
  • Call Center Representative Jobs Resume Location
  • Customer Service Representative Jobs Resume
  • Delivery Driver Jobs Resume Location
  • Warehouse Worker Jobs Resume Location
  • Account Executive Jobs Resume Location
  • Sales Associate Jobs Resume Location
  • Licensed Practical Nurse Jobs Resume Location
  • Company Driver Jobs Resume

Related posts

How To Answer "Do You Have Questions For Me?" (With Examples)

How To Answer “Do You Have Questions For Me?” (With Examples)

problem solving interview riddles

30 Phone Interview Questions (With Example Answers)

How To Answer "What Gets You Up In The Morning?"

How To Answer “What Gets You Up In The Morning?” (With Examples)

problem solving interview riddles

How To Answer “What Is Your Work Style” (With Examples)

  • Career Advice >
  • Interview Questions >

15 brain teaser interview questions and answers

problem solving interview riddles

In this blog post, we look at 15 brain teaser interview questions and answers to discover how your candidate thinks.

Job roles that require analytical or technical skills suit being asked brain teasers during the interview.

These questions test a number of skills, including logic, maths, critical thinking, creativity and the ability to problem-solve.

Brain teasers are different to the standard interview questions. Instead, the candidate's approach to how they get their answer is more important than the answer itself. - Even if they get the answer wrong.

A candidate who answers incorrectly but shows logic and analytical thinking will be better off than a candidate who doesn’t reveal their thought process.

Introduction: Why Do You Need Brain Teaser Interview Questions?

Brain teaser interview questions are a great way to test the problem-solving skills of a candidate. They are also a great way to see how well the person can think on their feet.

They are not for everyone, but for those who want to stand out, they can be a great asset.

These questions are used in interviews to assess the skills of a candidate. They are designed to test a range of these skills including problem solving and creativity.

People who solve these brain teaser questions correctly are able to think outside the box and come up with creative solutions. They also have the ability to think analytically and solve problems quickly.

As a recruiter, you get an insight into how your candidate thinks. You’ll understand how they approach problems and what their level of creativity is like.

Brainteasers - What are they?

Brainteasers are questions or puzzles that are designed to test a person's intelligence, creativity and problem-solving skills.

The term "brain teaser" is used to describe any puzzle, question, or riddle that requires thought and concentration to solve. Brain teasers can be classified into two groups:

  • Those that appear easy but have a deceptively difficult solution.
  • Those with a difficult solution for which the difficulty is immediately apparent.

These questions are designed to test the interviewee's mental agility. They can be used as a part of an interview process or to assess their suitability for a job.

While not always easy to answer, these questions require some thinking, but there is no one answer that is always right. That's why it is important for candidates to show their thought process when answering.

Brainteasers - How should candidates answer them

Some of the most popular brain teasers that interviewers ask candidates include:

What is heavier, a kilogram of feathers or a kilogram of bricks?

How many animals of each kind did Moses take on the ark?

What would you weigh on another planet where gravity is stronger than on earth?

Brainteasers are puzzles that test your creativity and critical thinking. They are used in interviews to assess how well you can think on your feet.

Brainteasers can be tricky, but there are a few ways to prepare for them:

- Practice logic puzzles

- Read the newspaper to stay up-to-date on current events

- Play games like chess or Scrabble

The 15 Most Popular Brain Teaser Interview Questions And How To Answer Them!

There are many ways to determine whether a candidate is qualified for a position. But these tests are a great way for employers to find out how well a candidate is equipped to do their job.

The following is a list of the top 15 most popular brainteaser interview questions.

Brain Teaser Question 1.

Which of the below is the most useful in guessing the number of people working in a 30 story building?

  •  The of cars in the car park.
  •  Number of people eating lunch in the cafeteria.
  •  Total number of people on the 11th floor.

While there is no correct answer specifically, you should be looking to see how the candidate answers this question.

How do they justify selecting their answer while they eliminate others? The answer is impossible without more data. Are candidates asking if the building is in an urban or suburban area?

They can then guess how many people would take public transport and how many would drive.

Otherwise, they can take an estimate on the number of people on one floor. -Allowing them to estimate the capacity of the building.

Brain Teaser Question 2.

In annual revenue, how much does Times Square Starbucks bring in?

Again, don’t look for specific answers. You should look to identify if the candidate can make justifiable assumptions on averages. One example can be the average cost of a ticket and traffic per hour, multiplied by 24, then 365.

Brain Teaser Question 3.

At exactly noon each day a scientist puts a bacteria in a petri dish. Every minute the bacteria divides in two. When it's at 1pm, the dish is full. What was the time when the dish was half full?

This question is not a trick question and doesn’t require the candidate to find out any extra information. The answer is quite simple. If every minute the bacteria doubles, and it’s full at 1pm. It would have been half full a minute earlier at 12:59pm.

Brain Teaser Question 4.

In one year, how many potatoes does McDonald's sell (in kg) in 1 year.

For this answer, candidates don’t need to know anything about Mcdonalds' potatoes. Rather, it’s how they approach the problem you should be mindful of.

To answer the question candidates should estimate the number of McDonald's restaurants in the region. Then to estimate the number of hash brown and fries orders a day, and the number of potatoes for each order. Then they should estimate the number of potatoes in a kilo.

Brain Teaser Question 5.

You have 50 black balls and 50 white balls and 2 buckets. How can you divide the balls into each bucket to maximise the probability of selecting a black ball at random? The ball can be chosen from either bucket at random.

For candidates to successfully answer this they should say:

There is just under a 75% chance of having a black ball chosen. It would be wise to put 1 black ball in 1 of the buckets and all the 99 balls in the other bucket.

With there being a 50% chance of selecting the bucket with one ball. Then there is a 100% chance of selecting a black ball.

Then there is also a 50% chance of selecting the bucket with 99 balls. With a 49.5% chance of selecting a black ball from this bucket.

The probability of selecting a black ball is worked out as: (50% % 100%) + (50%*49.5%) = 74.7%.

Brain Teaser Question 6.

If a car travels a distance of 60 miles at an average speed of 30 mph. How fast would the car need to travel the same 60 mile distance home to average 60 mph over the entire trip?

This is in fact a trick question! While most answers would say 90 mph the answer is something quite different.

At an average speed of 30mph, the first leg of the trip covers 60 miles. Equalling to the car travelling for 2 hours (60/30). For the car to average 60 mph over 120 miles, it would have to travel for exactly 2 hours (120/60). Since the car has already travelled for 2 hours, it’s impossible for it to average 60mph over the entire trip

Brain Teaser Question 7.

An interviewer presents you 3 envelopes. 

1 contains a job offer, the other 2 contain rejection letters. After you select 1 envelope, the interviewer shows you the contents of 1 of the other envelopes. -Which is a rejection letter. 

You are then given the option to stick with your original envelope or swap with the other envelope. Without knowing what your original envelope contains, should you switch?

To answer this question effectively, candidates should say yes.

This is because your original choice (envelope A) had a 1/3 chance of containing the offer letter. Leaving a 2/3 chance that the offer letter was either in envelope B or C.

By sticking with envelope A, your chance still remains 1/3.

Brain Teaser Question 8.

What is the angle between the hour-hand and minute-hand of a clock at 3:15?

At a quarter past the hour, the minute hand is exactly at 3:00. But the hour-hand has moved 1/4 of the way between 3:00 and 4:00.  Therefore 1/4 times 1/12 = 1/48 of the clock.  With the clock having 360 degrees, 360/48 = 7.5 degrees.

 Brain Teaser Question 9.

You are given a 3-gallon jug and a 5-gallon jug. How do you use them to get 4 gallons of liquid?

Fill the 5-gallon jug completely.  Pour the contents of the 5-gallon jug into the 3-gallon jug, leaving 2 gallons of liquid in the 5-gallon jug.

Next, dump out the contents of the 3-gallon jug and pour the contents of the 5-gallon jug into the 3-gallon jug.  At this point, there are 2 gallons in the 3-gallon jug.

Fill up the 5-gallon jug and then pour the contents of the 5-gallon jug into the 3-gallon jug until the 3-gallon jug is full.  You will have poured 1 gallon, leaving 4 gallons in the 5-gallon jug.

Brain Teaser Question 10.

You’ve got a 10 x 10 x 10 cube made up of 1 x 1 x 1 smaller cubes. The outside of the larger cube is completely painted red. On how many of the smaller cubes is there any red paint?

To answer this, it’s easier to think of how many of the cubes aren’t painted. The large cube is made up of 1000 small cubes. Of these, 8x8x8 of the inner cubes are not painted= 512 cubes.

Therefore, 1000-512= 488 cubes that have some paint.

Brain Teaser Question 11.

How would you test a calculator?

Again, like most of these brain teasers, there is no correct answer.

For the ideal answer candidates should say, they can test the functionality of the calculator's accuracy.

They can achieve this by evaluating whether the inputs provide their expected outputs. By testing the calculator's basic system functions. Like the power button, clear, etc to determine if there are system errors before anything else.

Brain Teaser Question 12.

You walk across a bridge and you see a boat full of people. Yet there isn’t a single person on board.

How is that possible?

These questions are designed to confuse candidates, by using conflicting words to create unexplainable scenarios.

Candidates must listen carefully to the wording of the question.

The answer to this question is “because their all married!” Suggesting more meaning to the use of the word ‘single’.

Brain Teaser Question 13.

Tracy’s mother has four children. One child is named April. The second one is May. The third is June. What‘s the fourth one’s name?

This is an illusion Brain Teaser. To answer this properly candidates must not get distracted by misleading details. The answer to this question is: “Tracy”.

The question tricks you into believing the fourth child is named “July” following the very obvious pattern of April-May-June. But it very clearly states the fourth name in the question.

Brain Teaser Question 14.

You are presented with two doors – one leading to Freedom, the other to Death. Each door has a guard, with one always lying and the other always telling the truth. For both the doors and the guards, you don’t know which one is which.

With just one question/request to one of the guards, how do you find the door of Freedom?

To answer this, candidates have to think logically and ask both guards the same question. “Please show me the door the other guard would lead me to, if I asked for the door of Freedom”. Both guards will point to the door of Death, so you take the other door.

This is because the truthful guard will show you what the lying one would do. Pointing to the death door.

The untruthful guard will show you the opposite of the truthful guard, pointing to the death door.

Brain Teaser Question 15.

What does this represent: “COF FEE”?

This is a letter-trick question. Candidates should examine many aspects of the letter to discover the meaning. The answer is “coffee break”.

How beneficial is it to ask brain teaser interview questions?

According to Google’s executive Laszlo Bock, Google has abandoned their brain teaser interview questions quoting, “They don’t predict anything.”

One of the downsides of asking brainteaser interview questions is that it can sour the candidate experience. Most candidates hate these questions. The ability to identify a great answer and link it to IQ is questionable and inconsistent.

However, on the flip side, these ‘trick’ questions allow recruiters to see the desired traits and behaviours in candidates. - Providing they can successfully recognise the skills in candidates' answers.

What do you think?

Is using brainteasers an effective method of identifying the candidate's logical thinking capacity? Or is it a time-waster, harming the candidate experience and the reputation of your brand?

Let us know in the comments below we’d love to hear what you think.

For more interviewing question tips see our blog post on: Strategic interview questions to ask candidates.

One comment on “15 brain teaser interview questions and answers”

[…] If you enjoyed this post on how to identify fake resumes, then you may love to read, “15 brain teaser interview questions and answers”. […]

Leave a Reply Cancel reply

Your email address will not be published. Required fields are marked *

Save my name, email, and website in this browser for the next time I comment.

QUICKLY HIRE YOUR PERFECT FIT

BrainEaser Logo

Ultimate Guide to Brain Teaser Interview Questions

Puzzle Pieces

Brain Easer’s guide to brain teaser interview questions

1. why brain teasers.

To understand how to tackle brain teasers at an interview, first understand what brain teasers are and why they are used.

What Are Brain Teasers

Brain teasers can be thought of broadly as types of puzzles that test problem-solving and critical thinking, and potentially other related skills such as logic, math, and creativity.

The Goal of Brain Teasers

An interviewer may ask a brain teaser to see how you would approach a problem or challenge, and to assess your critical thinking skills and how you think under pressure. Some brain teasers also test your ability to be flexible, creative, and adaptable.

Types of Interviews with Brain Teasers

You will most commonly encounter brain teaser interview questions in these industries or roles:

  • Quantitative finance – including institutional and prop trading, hedge funds, quant trading and modeling
  • Consulting – including management and strategy consulting
  • Engineering interviews – including software engineering and data science & analytics

Back in the day, Google and Microsoft were notorious for asking brain teasers, and every trader interview featured a brain teaser.

In recent years, brain teasers have largely fallen out of favor with interviewers, partly because they are now considered a poor way to assess on-the-job performance. But you may still encounter them for certain roles and firms, so read on and learn how to prepare!

2. How to Prepare

To prepare for brain teaser interviews, understand the types of brain teasers you are likely to encounter in an interview, and train with practice problems.

Types of brain teasers

Logic puzzles.

As the name suggests, logic puzzles test logical thinking and deduction, and usually feature multiple logical steps in which you use clues to rule out certain possibilities to arrive at a clever and tidy solution.

Often encountered in: finance interviews, some consulting interviews, and software engineering interviews.

Example : There are three boxes: one with two red widgets, one with two blue widgets, and one with one red and one blue widget. All three boxes are labeled incorrectly . What is the fewest number of widgets you have to take out in order to correctly re-label all the boxes? See the solution here

Math puzzles

There are many types of math puzzles, but interview brain teasers most commonly test probability because it can be relevant to a lot of technical jobs.

Often encountered in: technical finance interviews, software engineering, and data analytics interviews.

Example : In a best of 3 tennis match, the player that first wins 2 sets wins the match. For a 3-set tennis match, would you bet on it finishing in 2 or 3 sets? See the solution here

Estimation problems

Estimation problems test your ability to estimate some unknown value using limited information – typically solved by using some common knowledge and piecing together reasonable assumptions to arrive at a good estimate. These don’t usually have a single correct solution, the interviewer just wants to see if you can figure out what information you can bring in and apply to make a difficult problem a more tractable one.

Often encountered in: strategy/management consulting interviews (which often frame them in terms of “market-sizing questions”).

Example : How many people in Germany have iPhones? While there is technically a correct answer, there isn’t just one correct solution – the idea is to identify relevant pieces and put them together logically. One potential approach is to break down the problem into a couple of steps like this: (1) rough population of Germany, (2) smartphone ownership rates, (3) iPhone market share (you may have a general sense and a guess as to whether that share might be higher or lower in Germany).

Riddles use a cryptic clue to describe an item or concept that you must guess. These often require some lateral and creative thinking.

Our take: riddles do not belong in interviews, but that doesn’t stop interviewers from asking them! These are uncommon precisely because they are not good at testing critical thinking, but in some cases they can be suitable for testing more creative types of problem-solving, like forensic investigation. We would recommend not using these to prepare, unless you have reason to believe a particular interviewer or company asks these.

Example : What is black when you buy it, red when you use it, and gray when you throw it away? See the solution here

Prepare by practicing

Prepare with lots of good practice problems. The key to becoming proficient at pretty much anything is to practice. For brain teaser interviews, make sure to practice with suitably difficult questions that may get asked in an interview.

Don’t waste your time with super easy brain teasers (you’ll get them anyway), or the wrong kind of brain teaser.

If you just search up “brain teasers”, you might come across unsuitable brain teasers like this: How far can a squirrel run into the woods? The answer is: halfway – after that, the squirrel is running back out of the woods.

While this is a clever and cheeky brain teaser, it isn’t a brain teaser that you are likely to encounter in an interview.

Start with Brain Easer’s curated collection of interview brain teasers , which feature puzzles that are suitable for interviews, many of which are sourced from actual interviews . The logic and math puzzles with more than one insight or logical leap are particularly suitable for interviews, because an interviewee can make progress and demonstrate their critical thinking skills even if they don’t arrive at the final answer.

3. How to Approach the Interview

Get comfortable with how to approach solving brain teasers at an interview. These steps may help you effectively tackle other types of interview questions as well.

A. Clarify the question

You need to fully understand the question in order to come up with a good solution. Ask clarifying questions.

B. Talk through your reasoning

Always talk through your reasoning, for a number of helpful reasons:

  • You don’t necessarily need to correctly solve the brain teaser to do well, you just need to demonstrate how you would tackle a difficult problem. This is like getting partial credit in school for showing your work.
  • Talking through your reasoning can buy you some time to think, and for some people it also helps organize your thoughts.
  • This gives the interviewer an opportunity to steer you off the wrong track. You may misinterpret a piece of information or make an invalid assumption, and if the interviewer is able to jump in to correct your assumptions, you will waste less time.

C. Organize your solution

Remember that this is still an interview. While the brain teaser tests your critical thinking, you also should demonstrate other relevant skills to the job, such as organization, communication, and presentation.

So after you piece together the answer, organize your thoughts and clearly present the steps of your solution.

D. Impress the interviewer (optional)

If you’ve already solved the brain teaser, there is room to go above and beyond! If you have time left over, you can talk about variations and extensions on the brain teaser. Proactively solving a more difficult problem is likely to impress any interviewer.

Example : You and a friend play “first to 100”, a game in which you start with 0, and you each take turns adding an integer between 1 and 10 to the sum. Whoever makes the sum reach 100 is the winner. What is the winning strategy? See the solution here But after getting the right answer, you might add a restriction – neither player is allowed to add 11 minus what the other player just added – and show that there is still a (different) guaranteed winning strategy.

Naturally, this is difficult to do, so don’t worry if you are not able to.

Another way to engage meaningfully with the brain teaser after you’ve solved it: to discuss why it was interesting to solve and how it might relate to the type of work you would do on the job.

At the end of the day, brain teaser interview questions are like any other interview question – an opportunity to demonstrate why you are a good candidate for the job. Hopefully this guide helped take some of the pressure off solving the brain teaser itself and will allow you to put your best foot forward at the interview.

Solve some practice brain teasers now!

Leave a Reply Cancel Reply

Your email address will not be published. Required fields are marked *

Name  *

Email  *

Save my name, email, and website in this browser for the next time I comment.

Post Comment

Check out similar posts

Sicherman dice.

  • Difficulty - Hard , Probability

Who Scored How Many Points

  • Algebra , Difficulty - Hard , Simple Math

Double the Amount You Need to Do

  • Difficulty - Medium , Interview Brain Teasers , Simple Math

Stranded in the Wilderness Brain Teaser

  • Difficulty - Medium , Riddles and Lateral Thinking

problem solving interview riddles

How to Nail your next Technical Interview

You may be missing out on a 66.5% salary hike*, nick camilleri, how many years of coding experience do you have, free course on 'sorting algorithms' by omkar deshpande (stanford phd, head of curriculum, ik), help us with your details.

interviewkickstart dark logo

Top 20 Interview Puzzles for Software Engineers

Last updated by Utkarsh Sahu on May 30, 2024 at 05:59 PM | Reading time: 10 minutes

Puzzles are a realistic way of testing your lateral thinking in software engineer interviews. It shows the interviewer your real-world problem-solving and creative thinking skills. These puzzles are mostly popular among Tier-1 companies, which look for candidates with more than basic programming skills.

All the puzzles might not have a single solution. There may be multiple solutions, and the interviewer wants to see how you find the most efficient solution and how you think through the problem.

In this article, let’s discuss some popular puzzles asked in interviews. Here’s what we’ll cover:

Common Puzzle Questions

Coding puzzle questions.

  • Tips to Solve Puzzle Problems in Interviews
  • FAQs About Interview Puzzles for Software Engineers

Top 20 Puzzle Questions Asked in Interviews

Most of the puzzles asked in interviews are meant to test how well you can think “different.” They usually don’t have one correct answer to refer to. You can solve it using your own logic.

So, looking at 100s of puzzles online and checking their answers won’t be enough. You need to THINK on your own and actually develop an approach to solving these puzzles.

We’ve covered 20 sample puzzles to get your prep started.

Common puzzles are those types of puzzles where you might not require your programming skills. It’s based on your reasoning and logical skills.

Let’s look at some common puzzles asked in interviews.

1. Crossing the Bridge Puzzle

Four people need to cross a bridge. It’s nighttime and pretty dark. There’s only one flashlight; it’s dangerous to cross the bridge without one. The bridge can only support two people at a time. Each person will take a different amount of time to cross the bridge: 1 min, 2 mins, 7 mins, and 10 mins. What is the shortest possible time for all four people to cross the bridge?

Crossing the Bridge Puzzle

2. The Man in the Elevator Puzzle

A man who lives on the tenth floor of a building takes the elevator every day to go down to the ground floor to go to work or to go shopping. When he returns in the evening, he takes the elevator to the seventh floor and walks up the stairs to the tenth floor to reach his apartment. Why does he do this? Note that if it’s a rainy day, or if there are other people in the elevator, he goes to his floor directly. Also, he hates walking.

The Man in the Elevator Puzzle

3. Heaven or Hell Puzzle

You have two doors in front of you. One door leads to heaven, and the other to hell. There are two guards, one by each door. One guard always tells the truth, and the other always lies, but you don’t know who is who. You can only ask one question to one guard to find the door to heaven. What question would you ask?

Heaven or Hell Puzzle

4. Three Mislabeled Jars

You have three mislabeled jars. The first jar contains apples, the second contains oranges, and the third contains a mix of apples and oranges. You need to label the jars. You can pick as many fruits as you want from each jar. What is the least number of fruits you have to pick from each jar to label them correctly?

Three Mislabeled Jars

5. Gold Bar Cut Puzzle

You have hired someone to work for you for seven days, and you have a gold bar to pay him. You must give him a piece of gold every day. What is the least number of cuts you can make to the gold bar such that you can pay them 1/7th of it each day?

Gold Bar Cut Puzzle

6. Man Fell in Well Puzzle

A man fell in a well. The well is 30 meters deep. In a day, he can climb 4 meters, but he slips down 3 meters. How many days would he take to come out of the well?

Man Fell in Well Puzzle

7. Bag of Coins Puzzle

You have 10 bags full of infinite coins. But one bag is full of fake coins, and you can’t remember which one. You know that a genuine coin weighs 1 gram, and a fake coin weighs 1.1 grams. How do you identify the bag containing forged coins in minimum readings?

Bag of Coins Puzzle

8. Horses on a Race Track Puzzle

There are 25 horses and five race tracks. Find the fastest three horses among the 25 in the least number of races.

Horses on a Race Track Puzzle

9. Batteries Puzzle

There are eight batteries, but only four of them work. You have to use them for a flashlight, which needs two working batteries. What is the minimum number of battery pairs you need to test to ensure that the flashlight is turned on?

Batteries Puzzle

10. Birthday Cake Puzzle

A birthday cake has to be cut into eight equal pieces in exactly three cuts. Find a way to make this cut possible.

Birthday Cake Puzzle

11. Clock Angle Puzzle

If the time is 3:15 when you look at a clock, what’s the angle between the hour hand and the minute hand?

Clock Angle Puzzle

12. Tomato Soup Puzzle

You have a glass of tomato soup. You have one other empty glass of a different size and shape. You have to give the soup to two children. How would you divide the soup into two glasses so that both of them are satisfied that they have got an equal share of soup?

Tomato Soup Puzzle

Coding Puzzles are those types of puzzles where you require a programming language to solve. Let’s look at some Coding Puzzles asked in interviews .

1. Addition Puzzle

Add two numbers without using the addition operator.

2. Determine the If Condition Puzzle

What should be the if condition in the following code snippet to print ‘Hello World’?

if "condition"    printf ("Hello"); Else    printf("World");

3. Swap 2 Numbers Puzzle

Swap 2 numbers without using a third variable.

4. Print Numbers From 1 to N Puzzle

Print numbers from 1 to N without using any looping construct.

5. Print Semicolon Puzzle

Print a semicolon without using a semicolon in the program

6. Equal Integers Puzzle

Determine if 2 integers are equal without using comparison and arithmetic operators.

7. Minimum Number Puzzle

Given 2 numbers. Find the minimum number without using conditional statements or ternary operators.

8. Spotting a Truck Puzzle

The probability of spotting a truck on a highway in an hour is 0.999. What is the probability of spotting a track on that highway in 20 minutes?

Head over to the Learn and Problem pages for more.

3 Key Tips to Solve Puzzle Problems in Interviews

Yes, these puzzles are tricky. Following are 3 key tips that will ensure that you will not stumble during the interview:

Tips to Solve Puzzle Problems in Interviews

1. Clarify Everything Before you Start

Don’t jump into the solution, and do not make any assumptions. If any information seems missing, ask and clarify.

2. Explain Your Process

While solving the puzzle, explain your thought process to the interviewer. It allows your interviewer to see how you plan, think, reason, and solve complex problems under pressure. Always remember — these puzzles are more about showcasing your analytical skills than finding the right solution.

3. Provide a Solution to the Puzzle — One or More!

Use your reasoning and logical skills to deduce a solution to the puzzle, even if it’s not the correct solution. You can also discuss alternate methods. It’s more about your reasoning and deducing the solution, than finding the right solution.

FAQs on Puzzles for Software Engineers

Q. Do we get asked only programming puzzles in interviews ?

A. No. Interviewers test your ability to answer programming as well as common puzzles, which don't require programming skills.

Q. How to prepare for puzzle questions for an interview?

A. Practice multiple puzzles online, and try to solve them on your own without looking for answers. Interviewers will test your thinking abilities and insist more than getting the correct answer.

Q. Are puzzle interviews effective for employee selection?

A. Some of these calculations require significant approximations and come with unclear instructions regarding constraints. Due to their unconnected skill concentration, potential prejudice, and inability to accurately predict work performance, puzzle interviews are typically ineffective in the selection of employees.

Q. How many rounds of interviews do software engineers have?

A. There are generally four to five rounds of interviews for senior roles.

Preparing for a Tech Interview ?

Interview Kickstart can help you with coding puzzles and more! With IK, you get the unique opportunity to learn from and engage with FAANG tech leads and hiring managers .

Here’s what Aliya Mussina, IK alum and Software Engineer at Apple , has to say about IK’s program: Interview Kickstart's Program Met All My Expectations. Want to know more? Sign up for our FREE webinar .

problem solving interview riddles

Recession-proof your Career

Recession-proof your software engineering career.

Attend our free webinar to amp up your career and get the salary you deserve.

Ryan-image

Attend our Free Webinar on How to Nail Your Next Technical Interview

problem solving interview riddles

35 Amazon Leadership Principles Interview Questions

Top python interview questions for machine learning engineers, php mcqs with answers for backend developers, data mining mcqs: unearthing insights from big data, algorithm design and analysis mcqs for problem solvers, top 20 android interview questions for senior developers [2024], top python scripting interview questions and answers you should practice, complex sql interview questions for interview preparation, zoox software engineer interview questions to crack your tech interview, rubrik interview questions for software engineers, top advanced sql interview questions and answers, twilio interview questions, ready to enroll, next webinar starts in.

entroll-image

Get  tech interview-ready to navigate a tough job market

  • Designed by 500 FAANG+ experts
  • Live training and mock interviews
  • 17000+ tech professionals trained

problem solving interview riddles

The Most Common Interview Puzzles and How to Answer Them

Did you know that you have only seven seconds to make a good first impression in a job interview? One of the most daunting experiences during the job hunting process is the interview. Not only do you have to sell yourself to the company, but you also have to be prepared for any questions they might throw your way.

To help you prepare, we've compiled a list of some of the most common interview puzzles and how to answer them. With these tips in mind, you'll be one step closer to landing your dream job.

Keep reading and stop puzzling over whether or not you'll be prepared.

Table of contents

What is a puzzle question.

A puzzle question is a type of question that's designed to test your problem-solving skills. Puzzle questions are often used in job interviews, especially for technical positions.

Puzzle questions can be difficult to answer, but there are some strategies you can use to solve them. First, take a few deep breaths and relax.

Then, try to break the problem down into smaller pieces. Once you have a better understanding of the problem, start brainstorming possible solutions.

If you're still having trouble solving the puzzle, don't be afraid to ask for help from the interviewer or other people you know who are good at solving puzzles. With a little patience and effort, you should be able to find a solution to any puzzle question you're faced with.

Why Do Interviewers Ask Puzzle Questions?

Puzzle questions are often used in interviews as a way to gauge a candidate's problem-solving abilities. They can be tricky, and sometimes there is no single "right" answer. However, interviewers are looking to see how you approach and solve problems.

When it comes to hiring, employers are looking for a few key qualities in potential employees. First and foremost, they want to see that you have the skills and qualifications necessary to do the job. They'll also be looking for qualities like motivation, determination, and the ability to work well under pressure.

In general, employers are looking for three things in potential employees: technical skills, soft skills, and cultural fit. Technical skills are the specific abilities and knowledge required to do the job.

Soft skills are interpersonal skills that enable you to interact effectively with others. Cultural fit is a match between an applicant's values and the organization's values.

Puzzles can test all three of these areas. For example, a word puzzle can test an applicant's verbal reasoning ability, which is a form of intelligence that is important for many jobs. A spatial puzzle can test an applicant's ability to think creatively and come up with new solutions to problems. A puzzle that requires teamwork can test an applicant's ability to work well with others.

Thus, puzzles can be a helpful tool for employers in screening potential employees. However, it is important to keep in mind that no one type of assessment is perfect and that multiple measures should get used in order to get a complete picture of an applicant.

Here are some of the skills that puzzles can help test.

Logical Thinking

This is the most obvious skill that puzzles test. Do you have strong analytical and reasoning skills ?

Can you see patterns and relationships between things? You need to be able to think logically and systematically in order to solve most puzzles.

Some puzzles require you to think outside the box in order to find the solution. This is a great way to test your creative problem-solving skills.

Some puzzles can be quite challenging and require a lot of patience to solve. This is a good way to see how well you handle frustration and how long you are willing to persevere when faced with a difficult task.

There will be times when you need to focus intently on the puzzle in front of you in order to solve it. Can you stay focused on a task even when there are distractions?

Do you have an excellent attention span? This is a good way to see how well you can maintain focus and concentration under pressure.

Attention to Detail

Do you pay close attention to detail? Are you able to catch errors that others might miss?

The ability to pay attention to detail is a highly sought-after trait in many job descriptions. These employers are looking for candidates who can read and comprehend information, notice errors, and follow instructions correctly.

Problem-Solving Skills

Puzzles are often used as a means of testing problem-solving skills . This is because they require the individual to use a variety of skills in order to solve them.

These skills include creative thinking, logical reasoning, and pattern recognition. By testing these skills, employers can get a good idea of an individual's ability to solve problems.

How Do You Prepare for an Interview With Puzzles?

When you are preparing for an interview with puzzles, there are a few things that you can do to make sure that you are ready. First, you will want to familiarize yourself with the most common types of puzzles that get asked in interviews. This way, you will know what to expect and will be able to better prepare your responses.

Next, it is important to practice solving puzzles. This will help you to better understand how to approach them and will also give you a chance to see what types of clues or hints you may need in order to solve them.

Finally, make sure that you are comfortable with the format of the interview itself. This way, you will be less likely to get nervous and will be able to focus on solving the puzzles.

What Kind of Puzzles Are in Interviews?

When it comes to interview puzzles, there are a few different types that are commonly used. The most common type of puzzle is the brainteaser. Brainteasers test your problem-solving and critical-thinking skills.

They typically involve a mix of logic, math, and wordplay. Other common types of interview puzzles include riddles, logic problems, and pattern recognition exercises.

Riddles during Job Interviews

Riddles during job interviews are becoming increasingly popular. Many companies believe that they can help potential employers gauge a candidate's intelligence, problem-solving ability, and creativity. Riddles are often more straightforward than brainteasers, but they can still be challenging.

To solve a riddle, you need to be able to think logically and deductively. Pay close attention to the wording of the riddle, as this can often give you clues about the answer.

While there is no guarantee that you will be asked a riddle during your next job interview, it is always helpful to be prepared. Here are some example riddle questions and answers that may get used in a job interview.

Boat Filled With People

You see a boat filled with people. It has not sunk, but when you look again, you don't see a single person on the boat. Why?

What's the solution?

This is a classic riddle that has been around for centuries. It's one that can easily stump people; you might assume that the boat is filled with ghosts; however, there's a more logical explanation.

The answer to this riddle is that everyone on the boat is married. That's why you don't see a "single" person on board.

Egg vs. Concrete

How can you drop a raw egg onto a concrete floor without it breaking?

This riddle could leave you stuck. Your first thought will probably be that you can't drop a raw egg on the floor without breaking it.

However, remember, riddles test your ability to think outside of the box and creatively. Notice that in the riddle, it says "without it breaking."

The egg may crack, but the concrete floor won't break. This tests your ability to think critically and your listening skills.

Brainteasers

When it comes to interviewing, there are a few different types of questions you may be asked. One popular type is the brainteaser.

A brainteaser is a question that is designed to test your problem-solving and critical-thinking skills. They are often used in technical interviews but can also be found in other types of interviews as well.

To answer a brainteaser, you need to be able to think creatively and out of the box. Once you've identified the key information and relationships in the puzzle, you can start brainstorming possible solutions. It's often helpful to approach the problem from multiple angles and to consider all of the possible implications of your solution.

While they may seem daunting at first, with a little practice, you will be able to answer them with ease. In this article, we will go over some common brainteasers and how you can answer them.

While these questions may seem difficult, there are ways to approach them so that you can find the correct answer quickly and confidently.

How Would You Weigh a Plane Without Using Any Scales?

This brainteaser is a bit more complicated. However, there are several approaches you could take to answer it.

If you wanted to take a more scientific approach, your answer would focus on the facts. Here are some samples of a scientific approach:

One way would be to estimate the weight of the plane by its size and the materials it is made of. Another way would be to use the density of the air to calculate the weight of the plane.

To estimate the weight of the plane by its size, you would need to know the dimensions of the plane. You could then use these dimensions to calculate the volume of the plane. Once you have the volume, you could multiply it by the density of aluminum or other materials used in construction to get an estimate of the weight of the plane.

You could use the dimensions of the plane to calculate its volume. Once you have the volume, you could multiply it by the density of air to get an estimate of the weight of the plane.

However, maybe you know nothing about planes. Maybe you want to take more of a problem-solving approach. You could talk about the steps you would take to find the answer.

For example, you might research the solution online or ask a supervisor for help.

A more interpersonal approach would be to talk about checking company manuals and consulting coworkers or a supervisor. When you're answering this brainteaser, think about the job you're applying for and the skills you need.

This will help you give the best answer possible.

Michelle's Mom

Michelle's mom has four kids. Her first daughter is named April, followed by May and then June. What is the name of her fourth child?

The answer to this question is Michelle. However, many interview candidates will say July. This correct answer will show that the candidate has listening skills, uses logic, and is a quick thinker.

Strawberries and Bananas

There are three boxes in front of you. One box is labeled "STRAWBERRIES," another is labeled "BANANAS," and the third is labeled "MIXED."

Suddenly, you realize that all the labeling is wrong. You can only take out one piece of fruit from any one box without looking inside. How will you fix it?

Solving this brain teaser involves using listening skills, logic, and problem-solving skills. So what's the solution?

Note that the brainteaser states that all of the labels are wrong. That means the Mixed box will not be mixed. It will be either strawberries or bananas.

If you pull out a strawberry from the mixed box, you will move the strawberry label to that box. You then know that the banana box is still labeled incorrectly, but it can't be strawberries. That means it must be the mixed box, and the other box will be your banana box.

The Gallon Jugs

How do you measure four gallons of water using two container sizes that only come in three and five gallons?

The answer to this brainteaser requires you to use mathematical skills, logic, and problem-solving skills. While this one may leave you thinking for a minute, there are some possible solutions.

Here's one:

The first thing you want to do is fill up your three-gallon jug. You'll take that and pour it into your five-gallon jug.

Next, fill up your three-gallon jug again. Pour what you can into your five-gallon jug.

You will now have one-gallon left in the three-gallon jug. Pour out the five-gallon jug, then pour what's left in your three-gallon jug into your empty five-gallon jug.

Finally, fill up the three-gallon jug again and add it to the one-gallon that's already in your five-gallon jug. You now have four gallons in your five-gallon jug.

Math Problems

Answering math problems during an interview can be daunting, but it's important to remember that interviewers aren't looking for you to get the answer right. They're looking to see how you think and approach problem-solving.

With that in mind, here are some tips for answering math problems in an interview and some examples of math puzzles interviewers use.

Listen Carefully

Listen to the question carefully and make sure you understand what is being asked. If you're unsure, ask for clarification.

Take a Few Moments

Take a few moments to think about the problem before beginning to work on it. This will help you organize your thoughts and approach the problem in a logical way.

Go Step-by-Step

Work through the problem step-by-step, explaining your thought process as you go. This will help the interviewer understand how you're thinking about the problem and whether your approach is sound.

Ask for Help

If you get stuck, don't hesitate to ask for hints or clues from the interviewer. They'll likely be happy to provide guidance so that you can continue working on the problem.

Double Check Your Answer

Once you've arrived at a solution, take a moment to check it for accuracy and verify that it makes sense in the context of the problem. If it does, great! If not, try retracing your steps and see if you can identify where things went wrong.

Example One: How Would You Solve a Rubik's Cube?

This is a classic puzzle question that has been asked in interviews for years. To answer it, you need to demonstrate your ability to think logically and methodically.

The key is to break the problem down into smaller steps and then work through each one systematically. For example, you could start by solving one face of the cube, then move on to the next face, and so on.

Example Two: What Is the Next Number in This Sequence?

This type of question tests your mathematical skills and your ability to spot patterns. The best way to approach it is to look for any clues or patterns in the sequence that you can identify.

Once you've done that, try to work out what the underlying rule is that governs the sequence. From there, you should be able to work out what the next number will be.

Example Three: Two Trains Are Approaching Each Other from Opposite Directions

This is another classic puzzle question that commonly comes up in interviews. It tests your ability to think about problems in a three-dimensional way and to visualize complex scenarios.

The key here is not to get too bogged down in the details. Instead, focus on the big picture and try to work out a general solution that would apply in any situation.

Example Four: You Are on a Deserted Island with Nothing but a Knife

This is another question that tests your ability to think about problems in a three-dimensional way. The key here is to use your imagination and think about all the possibilities that are available to you.

For example, you could use the knife to make a fire, which would then allow you to signal for help. Or, you could use the knife to catch fish, which would provide you with food and water.

Example Five: There Are Three Points on a Line

This is a classic geometry problem that is often used in interviews. The key here is to visualize the problem in your head and then work out what information you need in order to solve it. Once you've done that, the answer should be relatively straightforward.

Example Six: You Are Given a Bag of Marbles

This is another classic puzzle question that tests your ability to think about problems in a three-dimensional way. The key here is to visualize the problem in your head and then work out what information you need in order to solve it. Once you've done that, the answer should be relatively straightforward.

Logical Questions

Logical questions are designed to test your problem-solving and critical-thinking skills. They are usually presented as a scenario or puzzle that you must figure out. For example, you may be asked to determine how many different ways there are to arrange a group of people given certain constraints.

To answer these types of questions, it is important to take your time and think through the problem carefully. Read the question carefully and identify all of the information that is given.

Then, start brainstorming possible solutions. Once you have a few potential solutions, try to eliminate any that are obviously incorrect. From there, you should be able to narrow down the possibilities and arrive at the correct answer.

If you get stuck, don't worry! These types of questions often have more than one correct answer.

The interviewer is looking to see how you think through problems and arrive at a solution, not necessarily whether or not you get the "right" answer.

The Classic '9 Dots' Puzzle

This is a classic puzzle that has been used in interviews for years. The challenge is to connect all nine dots using just four straight lines without lifting your pen from the paper.

The key to this puzzle is to think outside of the box. Most people assume that the lines must stay within the confines of the nine dots, but that is not the case.

If you draw lines that extend beyond the dots, you will be able to connect all nine dots with just four lines.

The Tower of Hanoi Puzzle

This puzzle is named after a temple in Vietnam where it is said that if you can solve it, you will achieve enlightenment.

The puzzle consists of three pegs and a set of disks of different sizes. The disks are stacked on one of the pegs, with the largest disk at the bottom and the smallest disk at the top.

The goal is to move all of the disks from one peg to another, following these rules:

You can only move one disk at a time.

You can only move a disk to an empty peg or onto a larger disk.

There are many different ways to solve this puzzle, but there is a specific strategy that will allow you to do it in the fewest number of moves.

Here's one way to do it:

  • Move the small disk from the first peg to the third
  • Move the medium disk from the first peg to the second peg
  • Move the small disk from the third peg to the second peg
  • Move the large disk from the first peg to the third
  • Move the small disk from the second peg to the first
  • Move the medium disk from the second peg to the third

Now you have successfully moved all disks from the first peg to the third while staying within the rules of the game.

The Bridge Crossing Puzzle

This puzzle is also known as the 'River crossing puzzle.' The challenge is to get all four people across a river using only one flashlight and two planks of wood. Each person can only cross the river once, and the flashlight can only be carried by one person at a time.

One potential solution is to have two people cross the river first, carrying the flashlight and one plank of wood. The two people can then use the plank of wood to help the other two people across.

The Knight's Tour Puzzle

The challenge here is to move a knight chess piece around an empty chess board, touching each square only once. This is actually quite difficult to do!

There are a few different ways that you could approach this problem. One is to use a brute force method, where you try every possible move until you find a solution. This is not very efficient, but it will eventually find a solution if one exists.

Another approach is to use something called backtracking. This is where you start with a knight on some square on the board and then try every possible move from there. If you ever get stuck, then you go back to the last square that you moved to and try a different move. You keep doing this until you either find a solution or determine that there is no solution for the given starting position.

Yet another way to solve this problem is with something called constraint satisfaction. This is where you have a list of constraints that the solution must satisfy, and you try to find a configuration of the knight that satisfies all of those constraints.

Which method do you think would be most efficient? Try implementing all three and see which one works best!

How to Answer Interview Puzzles

If you're preparing for an interview, you've likely heard that you should expect to be asked some brainteasers. While these types of questions can be daunting, there are some strategies you can use to prepare for them.

Before we get into how to answer interview puzzles, let's first understand what they are and why employers ask them. Interview puzzles are often used to test a candidate's problem-solving skills, creativity, and out-of-the-box thinking. They also give the employer a chance to see how the candidate responds under pressure.

With that being said, here are some tips on how to answer interview puzzles:

Take Your Time

Interviews are time-limited, but that doesn't mean you need to rush your thinking. Pause and think for a few moments when you see the question.

It's ok to ask the interviewer if they can give you a few minutes to formulate an answer. They will usually say yes, as it makes them look good too!

Be Creative

There is usually more than one correct answer to an interview puzzle, so don't be afraid to think outside the box. Employers want people who can problem-solve creatively and think in different ways.

Don't Get Stuck

If you really get stuck, explain to the interviewer what technique you are using and ask whether there are any other hints or suggestions they have. Most interviewers will be happy to help out as long as you are making earnest efforts yourself.

And don't forget: there is no harm in admitting that a certain puzzle is beyond your ability. Some interview puzzles cannot get solved with pure logic alone; they may require lateral thinking or even intuition.

Everybody has different strengths and abilities, so if it turns out that a particular puzzle is outside of yours, then so be it. You can still perform well in an interview by how you handle these situations!

As mentioned earlier, these types of questions are designed to test how you respond under pressure. It's important that you stay calm and don't let the puzzle stress you out.

The best way to prepare for interview puzzles is to practice them! There are many resources available online that provide sample questions and answers.

Familiarizing yourself with the types of puzzles employers may ask will help you feel more confident when it comes time for your interview.

Ace Your Interviews

Interview puzzles test a variety of skills, from problem-solving abilities to creative thinking. If you're looking for a job that requires these skills, then puzzles may be a good way to assess your fit for the role.

However, not all puzzles are created equal, so be sure to choose ones that test the specific skills you want to highlight. And if you get stuck, don't hesitate to ask for help - after all, teamwork is another important skill that employers value.

 Are you ready to continue testing your puzzling skills? Check out some unique puzzles and answers here. 

7 Things to Know About Vintage Jig Saw Puzzles

Seven things to know about vintage jigsaw puzzles. From the most popular themes to vintage jigsaw puzzle composition.

What Are 1 to 1, 2 to 2 ,And 3 to 3 Puzzles?

1 to 1, 2 to 2, and 3 to 3 puzzles are taxing, although, on the surface, they may appear simple. Connect six boxes without crossing the lines. Learn more here.

How to Value Vintage Jigsaw Puzzles

Vintage jigsaw puzzles are a great way to add a touch of art and nostalgia to your home. Here is a helpful guide on how to value them.

Puzzle Seek is about all things puzzles and games. So, if you seek a missing piece, a little history or a little fun we know you will find it here.

LEGAL INFORMATION

Puzzle Seek is a participant in the Amazon Services LLC Associates Program, an affiliate advertising program designed to provide a means for sites to earn advertising fees by advertising and linking to Amazon.com. Puzzle Seek also participates in affiliate programs with Clickbank, ShareASale, Cj and other sites. We are compensated for referring traffic and business to these companies.

© 2022 Copyright Puzzle Seek

7 Insane Brain Teasers You Could Actually Encounter in an Interview

woman thinking

You’ve made it through the first couple rounds of interviews, nailing questions like “Tell me a bit about yourself” and “Why do you want this job?”

But, in the final rounds for some types of roles (think very analytical or technical positions), you might encounter what could only be considered brain teasers. These kinds of questions aren’t to find out more about your previous experience or see if you fit into the company culture. They are used to test a number of your specific skills, including logic, math, critical thinking, creativity, and the ability to perform under pressure. And many times, your answer is actually irrelevant— it’s how you reached that answer that matters.

So, to help you brush up on your problem-solving capabilities, we’ve compiled the seven common types you could come across, as well as real-life examples of questions. But as you’re scrolling down and starting to stress that you could never (ever) even respond to these, remember that it’s all about your thought process. Hiring managers are much more interested in your problem solving skills than they are in actually knowing in how you would personally fight a bear.

Best of luck!

1. The “How Many [Things] Are There in [Location]?” Question

On the slight chance that your brain doubles as Google:

  • “How many gas stations are there in the U.S.?”
  • “How many cows are in Canada?”
  • “How many barbers are there in Chicago?”

2. The “How Many [Things] Could Fit in [Container]?” Question

File these under: “How and why would anyone ever know this?”

  • “How many ping pong balls could fit in a Boeing 747?”
  • “How many gallons of paint does it take to paint the outside of the White House?”
  • “How many trees are there in NYC’s Central Park?”

3. The “Do Some Quick Math” Question

In case your brain needed a really fast workout:

  • What is the sum of the numbers one to 100?
  • What is the angle between the hour-hand and minute-hand of a clock at [time]?
  • If I roll two dice, what is the probability the sum of the amounts is nine?

4. The “Why Is [Common Item] [the Way Common Item Is]” Question

These are also known as questions a four-year-old might ask that would also stump you:

  • “Why is a tennis ball fuzzy?”
  • “Describe the benefits of wearing a seatbelt.”
  • “Why are manhole covers round?”

5. The “Explain [Concept] to a [Difficult-to-Explain-Concept Person]?” Question

Otherwise known as, “explain your startup job to your grandmother at Thanksgiving” questions.

  • “Explain the internet to someone coming out of a 30-year coma.”
  • “Describe the color yellow to a blind person.”
  • “Teach me how to make an omelet.”

6. The “Solve This Mystery” Question

Oh, occasionally you’ll be asked to go detective and solve a mystery:

  • “A windowless room has three light bulbs. You are outside the room with three switches, each controlling one of the light bulbs. If you can only enter the room one time, how can you determine which switch controls which light bulb?” ( source )

Too easy? Here’s another:

  • “Four investment bankers need to cross a bridge at night to get to a meeting. They have only one flashlight and 17 minutes to get there. The bridge must be crossed with the flashlight and can only support two bankers at a time. The Analyst can cross in one minute, the Associate can cross in two minutes, the VP can cross in five minutes, and the MD takes 10 minutes to cross. How can they all make it to the meeting in time?” ( source )

7. The “How Would You Do Something Ridiculous” Question

And this last category is all about putting your creativity (and I guess, sometimes violence?) to the test:

  • “How would you fight a bear?”
  • “How would you kill a giraffe?”
  • “How would you test a calculator?”

Stumped on where to even begin? Fair enough. Muse writer Jeremy Schifeling has advice on actually solving these impossible brain teasers .

Oh, and do let me know on Twitter if I missed any good ones that you’ve been asked.

problem solving interview riddles

Top 20 Problem Solving Interview Questions (Example Answers Included)

Mike Simpson 0 Comments

problem solving interview riddles

By Mike Simpson

When candidates prepare for interviews, they usually focus on highlighting their leadership, communication, teamwork, and similar crucial soft skills . However, not everyone gets ready for problem-solving interview questions. And that can be a big mistake.

Problem-solving is relevant to nearly any job on the planet. Yes, it’s more prevalent in certain industries, but it’s helpful almost everywhere.

Regardless of the role you want to land, you may be asked to provide problem-solving examples or describe how you would deal with specific situations. That’s why being ready to showcase your problem-solving skills is so vital.

If you aren’t sure who to tackle problem-solving questions, don’t worry, we have your back. Come with us as we explore this exciting part of the interview process, as well as some problem-solving interview questions and example answers.

What Is Problem-Solving?

When you’re trying to land a position, there’s a good chance you’ll face some problem-solving interview questions. But what exactly is problem-solving? And why is it so important to hiring managers?

Well, the good folks at Merriam-Webster define problem-solving as “the process or act of finding a solution to a problem.” While that may seem like common sense, there’s a critical part to that definition that should catch your eye.

What part is that? The word “process.”

In the end, problem-solving is an activity. It’s your ability to take appropriate steps to find answers, determine how to proceed, or otherwise overcome the challenge.

Being great at it usually means having a range of helpful problem-solving skills and traits. Research, diligence, patience, attention-to-detail , collaboration… they can all play a role. So can analytical thinking , creativity, and open-mindedness.

But why do hiring managers worry about your problem-solving skills? Well, mainly, because every job comes with its fair share of problems.

While problem-solving is relevant to scientific, technical, legal, medical, and a whole slew of other careers. It helps you overcome challenges and deal with the unexpected. It plays a role in troubleshooting and innovation. That’s why it matters to hiring managers.

How to Answer Problem-Solving Interview Questions

Okay, before we get to our examples, let’s take a quick second to talk about strategy. Knowing how to answer problem-solving interview questions is crucial. Why? Because the hiring manager might ask you something that you don’t anticipate.

Problem-solving interview questions are all about seeing how you think. As a result, they can be a bit… unconventional.

These aren’t your run-of-the-mill job interview questions . Instead, they are tricky behavioral interview questions . After all, the goal is to find out how you approach problem-solving, so most are going to feature scenarios, brainteasers, or something similar.

So, having a great strategy means knowing how to deal with behavioral questions. Luckily, there are a couple of tools that can help.

First, when it comes to the classic approach to behavioral interview questions, look no further than the STAR Method . With the STAR method, you learn how to turn your answers into captivating stories. This makes your responses tons more engaging, ensuring you keep the hiring manager’s attention from beginning to end.

Now, should you stop with the STAR Method? Of course not. If you want to take your answers to the next level, spend some time with the Tailoring Method , too.

With the Tailoring Method, it’s all about relevance. So, if you get a chance to choose an example that demonstrates your problem-solving skills, this is really the way to go.

We also wanted to let you know that we created an amazing free cheat sheet that will give you word-for-word answers for some of the toughest interview questions you are going to face in your upcoming interview. After all, hiring managers will often ask you more generalized interview questions!

Click below to get your free PDF now:

Get Our Job Interview Questions & Answers Cheat Sheet!

FREE BONUS PDF CHEAT SHEET: Get our " Job Interview Questions & Answers PDF Cheat Sheet " that gives you " word-word sample answers to the most common job interview questions you'll face at your next interview .

CLICK HERE TO GET THE JOB INTERVIEW QUESTIONS CHEAT SHEET

Top 3 Problem-Solving-Based Interview Questions

Alright, here is what you’ve been waiting for: the problem-solving questions and sample answers.

While many questions in this category are job-specific, these tend to apply to nearly any job. That means there’s a good chance you’ll come across them at some point in your career, making them a great starting point when you’re practicing for an interview.

So, let’s dive in, shall we? Here’s a look at the top three problem-solving interview questions and example responses.

1. Can you tell me about a time when you had to solve a challenging problem?

In the land of problem-solving questions, this one might be your best-case scenario. It lets you choose your own problem-solving examples to highlight, putting you in complete control.

When you choose an example, go with one that is relevant to what you’ll face in the role. The closer the match, the better the answer is in the eyes of the hiring manager.

EXAMPLE ANSWER:

“While working as a mobile telecom support specialist for a large organization, we had to transition our MDM service from one vendor to another within 45 days. This personally physically handling 500 devices within the agency. Devices had to be gathered from the headquarters and satellite offices, which were located all across the state, something that was challenging even without the tight deadline. I approached the situation by identifying the location assignment of all personnel within the organization, enabling me to estimate transit times for receiving the devices. Next, I timed out how many devices I could personally update in a day. Together, this allowed me to create a general timeline. After that, I coordinated with each location, both expressing the urgency of adhering to deadlines and scheduling bulk shipping options. While there were occasional bouts of resistance, I worked with location leaders to calm concerns and facilitate action. While performing all of the updates was daunting, my approach to organizing the event made it a success. Ultimately, the entire transition was finished five days before the deadline, exceeding the expectations of many.”

2. Describe a time where you made a mistake. What did you do to fix it?

While this might not look like it’s based on problem-solving on the surface, it actually is. When you make a mistake, it creates a challenge, one you have to work your way through. At a minimum, it’s an opportunity to highlight problem-solving skills, even if you don’t address the topic directly.

When you choose an example, you want to go with a situation where the end was positive. However, the issue still has to be significant, causing something negative to happen in the moment that you, ideally, overcame.

“When I first began in a supervisory role, I had trouble setting down my individual contributor hat. I tried to keep up with my past duties while also taking on the responsibilities of my new role. As a result, I began rushing and introduced an error into the code of the software my team was updating. The error led to a memory leak. We became aware of the issue when the performance was hindered, though we didn’t immediately know the cause. I dove back into the code, reviewing recent changes, and, ultimately, determined the issue was a mistake on my end. When I made that discovery, I took several steps. First, I let my team know that the error was mine and let them know its nature. Second, I worked with my team to correct the issue, resolving the memory leak. Finally, I took this as a lesson about delegation. I began assigning work to my team more effectively, a move that allowed me to excel as a manager and help them thrive as contributors. It was a crucial learning moment, one that I have valued every day since.”

3. If you identify a potential risk in a project, what steps do you take to prevent it?

Yes, this is also a problem-solving question. The difference is, with this one, it’s not about fixing an issue; it’s about stopping it from happening. Still, you use problem-solving skills along the way, so it falls in this question category.

If you can, use an example of a moment when you mitigated risk in the past. If you haven’t had that opportunity, approach it theoretically, discussing the steps you would take to prevent an issue from developing.

“If I identify a potential risk in a project, my first step is to assess the various factors that could lead to a poor outcome. Prevention requires analysis. Ensuring I fully understand what can trigger the undesired event creates the right foundation, allowing me to figure out how to reduce the likelihood of those events occurring. Once I have the right level of understanding, I come up with a mitigation plan. Exactly what this includes varies depending on the nature of the issue, though it usually involves various steps and checks designed to monitor the project as it progresses to spot paths that may make the problem more likely to happen. I find this approach effective as it combines knowledge and ongoing vigilance. That way, if the project begins to head into risky territory, I can correct its trajectory.”

17 More Problem-Solving-Based Interview Questions

In the world of problem-solving questions, some apply to a wide range of jobs, while others are more niche. For example, customer service reps and IT helpdesk professionals both encounter challenges, but not usually the same kind.

As a result, some of the questions in this list may be more relevant to certain careers than others. However, they all give you insights into what this kind of question looks like, making them worth reviewing.

Here are 17 more problem-solving interview questions you might face off against during your job search:

  • How would you describe your problem-solving skills?
  • Can you tell me about a time when you had to use creativity to deal with an obstacle?
  • Describe a time when you discovered an unmet customer need while assisting a customer and found a way to meet it.
  • If you were faced with an upset customer, how would you diffuse the situation?
  • Tell me about a time when you had to troubleshoot a complex issue.
  • Imagine you were overseeing a project and needed a particular item. You have two choices of vendors: one that can deliver on time but would be over budget, and one that’s under budget but would deliver one week later than you need it. How do you figure out which approach to use?
  • Your manager wants to upgrade a tool you regularly use for your job and wants your recommendation. How do you formulate one?
  • A supplier has said that an item you need for a project isn’t going to be delivered as scheduled, something that would cause your project to fall behind schedule. What do you do to try and keep the timeline on target?
  • Can you share an example of a moment where you encountered a unique problem you and your colleagues had never seen before? How did you figure out what to do?
  • Imagine you were scheduled to give a presentation with a colleague, and your colleague called in sick right before it was set to begin. What would you do?
  • If you are given two urgent tasks from different members of the leadership team, both with the same tight deadline, how do you choose which to tackle first?
  • Tell me about a time you and a colleague didn’t see eye-to-eye. How did you decide what to do?
  • Describe your troubleshooting process.
  • Tell me about a time where there was a problem that you weren’t able to solve. What happened?
  • In your opening, what skills or traits make a person an exceptional problem-solver?
  • When you face a problem that requires action, do you usually jump in or take a moment to carefully assess the situation?
  • When you encounter a new problem you’ve never seen before, what is the first step that you take?

Putting It All Together

At this point, you should have a solid idea of how to approach problem-solving interview questions. Use the tips above to your advantage. That way, you can thrive during your next interview.

FREE : Job Interview Questions & Answers PDF Cheat Sheet!

Download our " Job Interview Questions & Answers PDF Cheat Sheet " that gives you word-for-word sample answers to some of the most common interview questions including:

  • What Is Your Greatest Weakness?
  • What Is Your Greatest Strength?
  • Tell Me About Yourself
  • Why Should We Hire You?

Click Here To Get The Job Interview Questions & Answers Cheat Sheet

problem solving interview riddles

Co-Founder and CEO of TheInterviewGuys.com. Mike is a job interview and career expert and the head writer at TheInterviewGuys.com.

His advice and insights have been shared and featured by publications such as Forbes , Entrepreneur , CNBC and more as well as educational institutions such as the University of Michigan , Penn State , Northeastern and others.

Learn more about The Interview Guys on our About Us page .

About The Author

Mike simpson.

' src=

Co-Founder and CEO of TheInterviewGuys.com. Mike is a job interview and career expert and the head writer at TheInterviewGuys.com. His advice and insights have been shared and featured by publications such as Forbes , Entrepreneur , CNBC and more as well as educational institutions such as the University of Michigan , Penn State , Northeastern and others. Learn more about The Interview Guys on our About Us page .

Copyright © 2024 · TheInterviewguys.com · All Rights Reserved

  • Our Products
  • Case Studies
  • Interview Questions
  • Jobs Articles
  • Members Login

problem solving interview riddles

  • Brain Teasers
  • Christmas Riddles
  • Funny Riddles
  • Interesting Riddles
  • Mathematical Riddles
  • Animal Riddles

Logical Puzzles

  • Mathematical Puzzles

Rubik's Cube

  • Aptitude-Puzzles
  • Top 100 Puzzles
  • Puzzles Quiz

Mathematical/Analytical Puzzle

  • Puzzle 1 | (How to Measure 45 minutes using two identical wires?)
  • Puzzle 2 | (Find the ages of daughters)
  • Puzzle 3 | (Calculate total distance travelled by bee)
  • Puzzle 6 | (Monty Hall problem)
  • Puzzle 16 | (100 Doors)
  • Puzzle 18 | (Torch and Bridge)
  • Puzzle | Set 35 (2 Eggs and 100 Floors)
  • Puzzle 12 | (Maximize probability of White Ball)
  • Puzzle 27 | (Hourglasses Puzzle)
  • Puzzle 17 | (Ratio of Boys and Girls in a Country where people want only boys)
  • Puzzle 29 | (Car Wheel Puzzle)
  • Puzzle 22 | (Maximum Chocolates)
  • Puzzle 28 | (Newspaper Puzzle)
  • Puzzle 33 | ( Rs 500 Note Puzzle )
  • Puzzle 39 | (100 coins puzzle)
  • Puzzle 44 | Girl or Boy
  • Puzzle 26 | (Know Average Salary without Disclosing Individual Salaries)
  • Puzzle 37 | (Maximum run in cricket)
  • Puzzle 32| (Completion of Task)
  • Puzzle 40 | (Find missing Row in Excel)
  • Four People on a Rickety Bridge
  • Puzzle | Man fell in well Puzzle
  • Puzzle | 50 red marbles and 50 blue marbles
  • Puzzle | Bag of Coins
  • Puzzle | Find the last ball to remain after the entire process
  • Puzzle | 10 identical bottles of pills
  • Hungry Worm and Old Tree Riddle | Puzzle
  • Puzzle 85 | Chain Link Puzzle
  • Interview Puzzle | The shopkeeper and the lady who made a purchase of Rs 200 with fake note
  • Puzzle | Snail and Wall
  • Puzzle 45 | Gem in Pockets
  • Pizza Puzzle
  • Puzzle 49 | King and his Elephants
  • Puzzle | Find the box which contains Box of 11 gm Cigarettes
  • Puzzle | How much he had initially?
  • Puzzle | Minimum planes to go around the world
  • Puzzle | Six colored cube
  • Puzzle 9 | (Find the fastest 3 horses)
  • Puzzle 5 | (Finding the Injection for Anesthesia)
  • Puzzle 4 | (Pay an employee using a gold rod of 7 units ?)
  • Puzzle 7 | (3 Bulbs and 3 Switches)
  • Puzzle 15 | (Camel and Banana Puzzle)
  • Puzzle 8 | (Find the Jar with contaminated pills)
  • Puzzle 10 | (A Man with Medical Condition and 2 Pills)
  • Puzzle 13 | (100 Prisoners with Red/Black Hats)
  • Puzzle 24 | (10 Coins Puzzle)
  • Puzzle 14 | (Strategy for a 2 Player Coin Game)
  • Puzzle 19 | (Poison and Rat)
  • Puzzle 20 | (5 Pirates and 100 Gold Coins)
  • Puzzle 31 | (Minimum cut Puzzle)
  • Puzzle 34 | (Prisoner and Policeman Puzzle)
  • Puzzle 36 | (Matchstick Puzzle)
  • Missionaries and Cannibals
  • Puzzle 41 | (Guess Color of Hat)
  • Puzzle 43 | Muddy Heads
  • Puzzle | Heaven and Hell
  • Puzzle | Mislabeled Jars
  • Puzzle | 8 balls problem
  • Puzzle 51| Cheryl’s Birthday Puzzle and Solution
  • Puzzle | Measure 4L using given 3 buckets
  • Puzzle | 3 Priests and 3 devils Puzzle
  • Puzzle | Farmer, Goat, Wolf and Cabbage
  • Puzzle | Water Jug Problem
  • Puzzle | Blind man and Pills
  • Puzzle | The Burning Candles
  • Birthday Puzzle
  • Puzzle | Rat and Poisonous Milk Bottles
  • Measuring 6L water from 4L and 9L buckets
  • Puzzle | Six Houses P, Q, R, S, T, and U
  • Melting Candles | Puzzle
  • Puzzle 47 | Red Hat vs Blue Hat
  • Puzzle | Find the Culprit
  • Puzzle 54 | Fill the Jug
  • Puzzle 46 | Rich or Poor
  • Puzzle | Light all the bulbs
  • Puzzle | Distribute the Water
  • Puzzle 50 | The Boat Wreckage
  • Puzzle | Weight of Heavy Ball
  • Puzzle | Guess the bit string

Arrangement Puzzles

  • Puzzle 11 | ( 1000 Coins and 10 Bags )
  • Puzzle 23 | (Days of month using 2 dice)
  • Puzzle 38 | (Tic Tac Toe Puzzle)
  • Puzzle 30 | (Last Palindrome Date Before 10/02/2001)
  • Puzzle 42 | (Placing the numbers)
  • Puzzle | 10 Balls in 5 Lines
  • Puzzle | (Round table coin game)
  • Puzzle | Place numbers 1 to 9 in a Circle such that sum of every triplet in straight line is 15

Shape based Puzzles

  • Puzzle 21 | (3 Ants and Triangle)
  • Puzzle | 3 cuts to cut round cake into 8 equal pieces
  • Puzzle 25 | (Chessboard and dominos)
  • Puzzle | Three Squares
  • Puzzle | Maximum pieces that can be cut from a Circle using 6 straight lines
  • Puzzle | Splitting a Cake with a Missing Piece in two equal portion
  • Puzzle 48 | Rectangular Cardboard
  • Puzzle | Dividing a Square into N smaller squares
  • Algorithm to solve Rubik's Cube
  • Rubik's Cube Facts and Questions
  • 20 Challenging Brain Teasers with Answers

Crossword Puzzle

  • Crossword Puzzle Of The Week #1 (for DSA)
  • Crossword Puzzle Of The Week #2 (for Computer Science and Applications)
  • Crossword Puzzle Of The Week #3 (for Database and Queries)
  • Crossword Puzzle Of The Week #4 (for Object Oriented Programming)
  • Crossword Puzzle Of The Week #5 (for Operating System)

Top 100 Puzzles Asked In Interviews

Puzzles are asked in Interviews to check our problem-solving skills. Here we are providing you with the top 100 puzzles that are asked in Interviews. The main aim of puzzles is entertainment. Puzzles can be of many types:- picture puzzles, logical puzzles, mathematical puzzles, etc. It is very helpful for the development of young minds and growth. The solver of a puzzle must arrive at the correct answer, or answers, by thinking or putting pieces together in a logical way.

Below is the list of the top 100 puzzles that are asked in Interviews:

Logical Puzzles:

1 FAANG,
2 , ,
3 ,
4 ,
5 ,
6
7 ,
8 ,
9
10
11
12
13 ,
14
15
16 ,
17 ,
18 , Simence
19 , Whatsapp, Singapore math Olympic
20 The Access Group (UK),
21
22
23 Mentor Graphics
24
25 , ,
26
27
28
29 Faang
30
31
32 ,
33
34 ,
35
36 , inflame
37

Mathematical And Analytical Puzzles:

S.No.

Puzzle Title

Asked in 

1 ,
2
3 ,
4
5 ,
6
7
8
9 ,
10 ,
11
12 ,
13 ,
14 CAT,
15
16 , .
17 FAANG
18 Reflexis Systems
19
20 Jumbotail,
21
22 , , J
23
24
25
26
27
28 ,
29 Leetcode
30
31 UK university interview
32 ,
33
34
35
36
37
38 ,
39
40
41 , , ,
42 , ,

Arrangement Puzzles:

S.No.                                           Puzzle Title Asked in 
1 ,
2

3

4

Kirloskar Brothers,

5

6
7

8

ElectrifAi

9 ,

Shape Puzzles:

S.No.

Puzzle Title

Asked in 
1. , ,
2

3

4

5

, ,

Other Puzzles:

S.No.

Puzzle Title

Asked in 
1.

2

FAANG

3

,

4

5

Conclusion:  

Puzzles are very helpful to improve logical thinking day by day.  Start practicing puzzles to crack interviews. These days puzzles are asked too frequently in top product-based companies. By solving the above puzzles you are enough prepared to solve the similar type of puzzles asked in your upcoming interviews.

Please Login to comment...

Similar reads, improve your coding skills with practice.

 alt=

What kind of Experience do you want to share?

Job Interview Tips

  • Customer Service
  • Graduate Jobs
  • Information Technology
  • Cover Letter
  • Interview Process
  • Interview Type
  • Social Media
  • What Should I Wear
  • Job Related
  • Personal Brand
  • Previous Job
  • Problem Solving
  • Weakness and Strength
  • Questions to Ask
  • Interview Skills
  • Common Mistakes
  • Salary Negotiation

Sample Logic Puzzles Given at Job Interviews

Companies today are doing whatever they can to try to hire only the best and the brightest. To accomplish this, some companies – Most notably, Microsoft and other IT companies – have added logic puzzles to their standard interview questions. Logic puzzles are designed to provide the company with insight into how well the applicant can solve problems and think outside the box for solutions. They are a great on-the-spot test of intelligence and provide almost as much information as an applicant’s work history. Below are some of the logic puzzles given at job interviews and their answers.

Sample Job Interview Logic Puzzles

Question: You have 8 jars of the same size and shape. Seven of the jars weigh 5 ounces while the eighth jar weights 6. You have a scale you can use to measure the jars but you can only measure twice. How do you find out which is the heavier jar?

Answer: You take two jars away. Put 3 jars on one side of the scale and 3 jars on the other side. Measure. If neither of them are heavier, the jar must have been in one of the two you took away. Measure and you will find the jar. If one of the scales did tilt, take 1 jar away of the remaining 3 and measure the two remaining jars on each side of the scale. If one tilts, that is the heavier jar. If neither tilts, the one remaining jar must be the heavy one.

Question: A snail is at the bottom of a 30 foot well. Every hour the snail is able to climb up 3 feet, then immediately slide back down 2 feet. How many hours does it take for the snail to get out of the well?

Answer: 28 hours. The snail is travelling at one foot per hour, except on the 28 th hour the snail has already reached the top of the well, so it will not slide down the 2 feet.

Question: There are 3 light bulbs in a hidden room and 3 switches outside the room that correspond to those light bulbs. You do not know which switch affects which bulb and you cannot see inside of the room. You are allowed to go inside of the room only one time. How do you find out which switch corresponds to which bulb?

Answer: Turn on two switches and wait for a while. Then turn off one switch and go inside the room. The bulb that is still on corresponds to the switch that is still on. Touch the remaining bulbs. The hotter bulb is the switch that you turned off, and the remaining bulb is the switch that you never turned on.

Answering Logic Puzzles

Logic puzzles can be difficult. The key to answering logic puzzle interview questions is not just in solving the problem, but in using sound reasoning. If you don’t know an answer, try your best to explain your thought process. Even if you are wrong, you should be able to impress the interviewer at most job interviews.

Take Away Interview Tips

  • Try to think outside the box.
  • Don’t rush to your answer, and don’t start explaining an answer you haven’t completely thought through.
  • Always explain your though process.

Related Posts

  • If They Offer Coffee, Should I Say Yes at the Job Interview? May 9, 2016
  • Interview Question: Describe Your Ideal Company December 26, 2011
  • Interview Question: What is Your Sense of Our Corporate Environment? December 23, 2011
  • What Would You Do if You Needed Help With a Task? December 21, 2011

MORE JOB INTERVIEW TOPICS

  • Achievements
  • Behavioural
  • Communication
  • Company Research
  • Difficult Questions
  • Interesting
  • Outside Work
  • Personal Skills
  • Presentation Skills
  • Prioritisation
  • Question Type
  • Quitting Your Current Job
  • Repetitive Tasks
  • Scheduling an Interview
  • Time Pressure
  • Tips and Interview Process
  • Uncategorized
  • Unemployment

ACE YOUR JOB INTERVIEW

problem solving interview riddles

Better resumes & cover letters

AI Resume Builder

Resume Checker

AI Cover Letters

Resume Summary Generator

Resume Bullet Generator

Resume Skills Generator

Tailor your resume to jobs

Job Tailored Resumes

Resume Keyword Scanner

Job Keyword Finder

Organize your search

Job Tracker

Contact Tracker

Interview Tracker

Job Search Metrics

Speed up your search

Job Application Autofill

Chrome Extension

  • For Organizations
  • University Career Centers
  • Career Coaches
  • Workforce Development
  • Outplacement
  • Affiliate Program

huntr logo

Targeted Resume

Beautiful, perfectly job-tailored resumes designed to make you stand out, built 10x faster with the power of AI.

  • For Organizations University Career Centers Career Coaches Bootcamps Workforce Development Outplacement Affiliate Program
  • Sign up for free

Interview Questions

50 Interview Questions About Problem Solving (With Answers)

Problems often arise at work. Can you solve them? Here are 50 interview questions about problem solving to know about.

March 22, 2024

Working in a company, you’ll be tasked with projects to work on that solve problems. Maybe you’ll need to solve a problem to help achieve a specific goal or create a solution to a problem that helps your business’ customers. Either way, problem solving is an important skill in the workplace. This post will include 50 interview questions about problem solving with answers to help you prepare for these types of interviews.

Get more interview questions. Sign up for Huntr to access interview questions tailored to the job you're applying for.

What is problem solving?

Problem solving is a critical soft skill that involves identifying, analyzing, and resolving issues or obstacles effectively and efficiently. It encompasses a series of steps including understanding the problem, brainstorming potential solutions, evaluating the pros and cons of each option, and implementing the most viable solution. Problem solving is not only about finding quick fixes but also about foreseeing potential challenges and addressing them proactively. It requires creativity, critical thinking, and the ability to remain calm under pressure, making it a highly valued skill in personal and professional contexts alike. Whether it's navigating interpersonal conflicts, troubleshooting a technical issue, or overcoming logistical hurdles, adept problem solvers are equipped to tackle a wide array of challenges, turning obstacles into opportunities for growth and improvement.

problem solving

Why problem solving is important in the workplace

1. enhances adaptability.

In today's fast-paced work environment, the ability to engage in effective problem solving is crucial. It allows employees to adapt quickly to changes, challenges, and unexpected situations. By developing strong problem-solving skills, individuals can analyze issues, identify solutions, and implement strategies efficiently, ensuring the smooth operation of workplace processes.

2. Promotes Innovation

Problem-solving skills are the backbone of innovation within the workplace. When employees are equipped to tackle challenges creatively, they can generate novel ideas and solutions that drive the company forward. This skill not only aids in overcoming immediate obstacles but also contributes to the long-term development and competitiveness of the organization.

3. Improves Team Collaboration

Effective problem solving often requires a collaborative effort. When team members possess strong problem-solving skills, they can work together more effectively, sharing insights, and combining their expertise to find the best solutions. This not only leads to better outcomes but also fosters a culture of cooperation and mutual respect among colleagues, enhancing overall workplace morale and productivity.

solving hard problems

5 Tips for Answering Problem-Solving Interview Questions

Problem-solving is a critical skill that employers look for in candidates across various industries. Demonstrating your ability to tackle challenges effectively can set you apart from other applicants. Here are five tips to help you showcase your problem-solving skills during an interview:

1. Use the STAR Method

Structure your responses using the Situation, Task, Action, and Result (STAR) method. Start by describing a specific situation or problem you encountered, outline the task you needed to accomplish, detail the actions you took to address the problem, and finish with the results of your efforts. This method helps you deliver a concise and compelling story that highlights your problem-solving prowess.

2. Highlight Your Analytical Skills

Employers want to see that you can analyze a situation and come up with a logical solution. When answering questions, talk about how you break down complex problems into manageable parts. Mention any tools or techniques you use, such as SWOT analysis, root cause analysis, or critical thinking strategies, to diagnose issues and develop solutions.

3. Show Adaptability

Problem-solving often requires flexibility and the ability to adapt to changing circumstances. Share examples of how you've had to adjust your approach when faced with new information or unforeseen challenges. This demonstrates your resilience and willingness to pivot your strategy to achieve the best outcome.

4. Emphasize Collaboration

Many problems are too complex for one person to solve alone. Talk about times when you've collaborated with others to tackle a challenge. Highlight your ability to listen to different perspectives, integrate feedback, and work as part of a team to find innovative solutions. This shows that you value diverse input and can leverage collective intelligence to overcome obstacles.

5. Reflect on Lessons Learned

Finally, don't shy away from discussing problems that didn't go as planned. Reflecting on what didn't work and what you learned from those experiences can be just as powerful as showcasing your successes. It demonstrates self-awareness, accountability, and a commitment to continuous improvement. Be sure to end on a positive note by explaining how these lessons have enhanced your problem-solving abilities moving forward.

By following these tips and preparing thoughtful, specific examples, you'll be able to convincingly demonstrate your problem-solving skills in your next interview. Remember, it's not just about the problems you've solved, but how you've solved them that will impress potential employers.

working towards a solution

1. Can you describe a situation where you had to solve a difficult problem at work?

In a previous role, our company faced a significant customer dissatisfaction issue due to a software glitch that caused frequent service disruptions. As the project manager, I had to lead a cross-functional team to identify the root cause, develop a solution, and implement it swiftly to restore customer satisfaction. It involved collaborating with the IT team, customer support, and product development to conduct a thorough analysis, prioritize fixes, and communicate transparently with customers to manage expectations and regain trust.

2. How do you approach complex problems in your professional life?

I approach complex problems by first breaking them down into smaller manageable components to gain clarity on the issue. Then, I gather relevant data, analyze trends, and identify patterns to understand the underlying factors contributing to the problem. Collaborating with colleagues from diverse backgrounds helps gain different perspectives and insights. I also prioritize actions based on urgency and impact, continuously evaluate progress, and adapt strategies as needed to achieve effective solutions.

3. What is your process for identifying the root cause of a problem?

My process for identifying the root cause involves asking probing questions, conducting thorough research, gathering data and feedback from stakeholders, analyzing trends and patterns, and using problem-solving techniques such as root cause analysis (RCA) or the "5 Whys" method. I focus on understanding the systemic issues rather than just addressing symptoms to ensure long-term solutions.

4. Can you give an example of a creative solution you've implemented to address a challenging issue?

In a project where budget constraints were hindering progress, I proposed implementing a lean approach by prioritizing essential features, streamlining workflows, and optimizing resource allocation. This creative solution allowed us to deliver key functionalities within budget and timeline constraints while maintaining quality and stakeholder satisfaction.

5. How do you prioritize problems that need to be solved?

I prioritize problems based on their impact on organizational goals, customer experience, urgency, and resource availability. I categorize issues into immediate, short-term, and long-term priorities, considering the potential risks and benefits of solving each problem. Regularly reassessing priorities ensures alignment with evolving business needs and strategic objectives.

6. What tools or methods do you typically use for problem-solving?

I utilize various problem-solving tools and methods depending on the complexity of the issue, including brainstorming sessions, fishbone diagrams, SWOT analysis, process mapping, data analytics, and project management software. Collaborative platforms and communication tools also facilitate effective teamwork and information sharing during problem-solving processes.

7. How do you involve others in the problem-solving process?

I involve others in the problem-solving process by fostering a culture of collaboration, open communication, and shared responsibility. This includes seeking input and perspectives from team members, encouraging diverse ideas and approaches, delegating tasks based on strengths, providing support and resources, facilitating brainstorming sessions or workshops, and recognizing contributions to promote a sense of ownership and accountability in problem-solving efforts.

8. Can you share an experience where you had to solve a problem under a tight deadline?

In a previous project, we faced a critical technical issue just days before a major product launch. The issue threatened the functionality of key features, risking customer satisfaction and revenue. To solve it, I organized a rapid-response team, conducted round-the-clock troubleshooting, prioritized tasks based on impact, and delegated responsibilities efficiently. Despite the tight deadline, we collaborated effectively, leveraged expertise from various departments, and implemented a temporary workaround to ensure a successful product launch. Post-launch, we continued refining the solution for a permanent fix.

9. How do you ensure that a solution is both effective and efficient?

I ensure that a solution is both effective and efficient by aligning it with predefined objectives and success criteria, conducting thorough analysis and testing, seeking feedback from stakeholders, and evaluating its impact on key performance indicators (KPIs) such as cost, time, quality, and customer satisfaction. Continuous monitoring and iteration allow me to optimize the solution's efficiency while maintaining its effectiveness over time.

10. What is the biggest problem you have solved in your career?

One of the biggest problems I solved in my career was optimizing a supply chain process for a multinational company. The challenge involved reducing lead times, improving inventory management, and enhancing supplier relationships to meet growing customer demands and reduce costs. By implementing data-driven strategies, process improvements, and cross-functional collaboration, we achieved significant improvements in efficiency, cost savings, and customer service levels, contributing to the company's overall success.

11. How do you deal with uncertainty when solving problems?

When faced with uncertainty, I adopt a structured approach by gathering relevant information, conducting scenario analysis, identifying potential risks and opportunities, and developing contingency plans. I also leverage past experiences, seek input from subject matter experts, and remain adaptable and open to alternative solutions. Clear communication, ongoing evaluation, and agile decision-making help navigate uncertainty effectively and mitigate potential impacts.

12. Can you describe a time when you had to solve a problem without all the necessary information?

In a project where critical data was unavailable due to technical issues, I had to make decisions and implement solutions based on limited information. To address this challenge, I leveraged available data, conducted qualitative analysis, consulted with experts, and communicated transparently with stakeholders about the uncertainties and risks involved. Despite the constraints, we managed to implement a temporary solution and later refined it based on additional insights and data.

13. How do you measure the success of a solution?

I measure the success of a solution by defining clear metrics and KPIs aligned with the problem's objectives and desired outcomes. This includes assessing factors such as cost savings, time efficiency, quality improvement, customer satisfaction, revenue generation, and impact on organizational goals. Regular performance monitoring, data analysis, feedback collection, and stakeholder evaluations help track progress, identify areas for improvement, and ensure that the solution delivers measurable benefits.

14. What do you do if your initial solution to a problem fails?

If my initial solution to a problem fails, I conduct a thorough analysis to understand the root cause of the failure, gather feedback from stakeholders, and reassess the problem and its context. I then iterate and refine the solution, incorporating lessons learned, alternative approaches, and additional insights to develop a more effective and robust solution. Continuous improvement, flexibility, and resilience are key elements in overcoming setbacks and achieving successful outcomes.

15. How do you adapt your problem-solving strategies in a rapidly changing environment?

In a rapidly changing environment, I adapt my problem-solving strategies by staying informed about industry trends, technological advancements, and market dynamics. I prioritize agility, collaboration, and innovation, regularly reassessing priorities, adjusting strategies, and leveraging emerging tools and methodologies to address evolving challenges effectively. Flexibility, quick decision-making, and a proactive approach to change enable me to navigate uncertainties and drive successful problem resolution.

16. Can you give an example of a time when you had to use data analysis for problem-solving?

In a marketing campaign analysis project, we noticed a significant drop in conversion rates for a particular segment of our target audience. To identify the underlying issue, I conducted a detailed data analysis using customer demographics, behavior patterns, and campaign performance metrics. By correlating data points and conducting A/B testing, we discovered that the drop in conversion was due to a mismatch between the messaging in the campaign and the preferences of that specific audience segment. This data-driven insight allowed us to adjust our marketing strategy effectively and improve conversion rates.

17. How do you differentiate between symptoms and root causes of a problem?

To differentiate between symptoms and root causes of a problem, I employ techniques such as the "5 Whys" method, root cause analysis, and process mapping. By asking probing questions and digging deeper into the factors contributing to the problem, I can identify underlying causes rather than just addressing surface-level symptoms. This approach ensures that solutions target the root of the problem, leading to more effective and sustainable outcomes.

18. Can you discuss a time when you had to solve a problem collaboratively with a team?

In a product development project, we encountered challenges related to compatibility issues between different software components, leading to performance issues and customer complaints. To address this, I facilitated collaborative problem-solving sessions with cross-functional teams, including developers, QA testers, and product managers. Through open communication, brainstorming, and sharing of expertise, we identified the root causes, developed a coordinated action plan, and implemented solutions that resolved the compatibility issues, improving product performance and customer satisfaction.

19. How do you handle feedback on your problem-solving efforts?

I value feedback as an opportunity for growth and improvement. When receiving feedback on my problem-solving efforts, I actively listen, seek to understand perspectives, and appreciate constructive criticism. I use feedback to reflect on my approach, identify areas for enhancement, and incorporate suggestions into future problem-solving endeavors. Embracing feedback fosters continuous learning, enhances problem-solving skills, and contributes to achieving better outcomes.

20. What role does critical thinking play in your problem-solving process?

Critical thinking is integral to my problem-solving process as it enables me to analyze situations objectively, evaluate information, identify patterns, and make informed decisions. By applying logical reasoning, sound judgment, and evidence-based analysis, I can discern between relevant and irrelevant data, assess risks, consider alternative solutions, and anticipate potential outcomes. Critical thinking enhances problem-solving effectiveness by promoting thoroughness, accuracy, and strategic decision-making.

21. How do you stay motivated when faced with a complex problem?

When faced with a complex problem, I stay motivated by breaking the problem down into manageable tasks, setting realistic goals, and focusing on incremental progress. I maintain a positive mindset, leverage my problem-solving skills and past successes, seek support and collaboration from colleagues or mentors, and celebrate small victories along the way. Staying organized, maintaining a clear vision of the desired outcome, and reminding myself of the impact of solving the problem keep me motivated and determined to overcome challenges.

22. Can you describe a situation where you had to change your approach to solve a problem effectively?

In a project involving customer feedback analysis, my initial approach focused solely on quantitative data analysis to identify trends and patterns. However, I realized that the qualitative aspect of customer feedback, such as sentiments and specific comments, provided valuable insights that were missed in the quantitative analysis alone. To address this, I changed my approach by integrating qualitative data analysis techniques, such as sentiment analysis and thematic coding, into the process. This holistic approach led to a more comprehensive understanding of customer perceptions and facilitated more targeted problem-solving strategies.

23. How do you balance the need for quick solutions with the need for thorough problem-solving?

Balancing the need for quick solutions with thorough problem-solving involves prioritizing tasks based on urgency and impact. For urgent issues requiring immediate resolution, I focus on quick, interim solutions to address critical aspects and mitigate immediate risks. Simultaneously, I allocate time and resources for in-depth analysis, root cause identification, and long-term solutions to prevent recurrence and optimize outcomes. Effective time management, clear prioritization, and strategic decision-making enable me to strike a balance between speed and thoroughness in problem-solving.

24. Can you discuss a time when you had to solve a problem with limited resources?

In a project where budget constraints limited our ability to hire additional staff, I faced the challenge of increasing operational efficiency without increasing costs. To address this, I conducted a thorough analysis of existing processes, identified inefficiencies, and proposed automation solutions using available software tools. By leveraging technology and optimizing workflows, we were able to streamline operations, reduce manual tasks, and improve productivity without exceeding the budget.

25. What strategies do you use to prevent problems from occurring in the first place?

Proactive problem prevention involves risk assessment, continuous monitoring, and implementing preventive measures. I regularly conduct risk assessments to identify potential issues, develop contingency plans, and implement controls to mitigate risks. Additionally, I emphasize continuous improvement, encourage open communication within teams, and promote a culture of accountability and learning from past experiences to prevent recurring problems.

26. How do you communicate complex problems (and solutions) to stakeholders who may not be familiar with the details?

When communicating complex problems and solutions to stakeholders, I use a structured approach that involves breaking down technical information into easily understandable concepts. I focus on highlighting the impact, benefits, and relevance of the problem and proposed solutions to the stakeholders' interests and objectives. Utilizing visual aids, such as charts, graphs, and diagrams, helps clarify complex information, facilitate discussions, and ensure stakeholders grasp key points effectively.

27. Can you share an example of a time when you solved a problem that significantly benefited your organization?

In a cost optimization initiative, I identified inefficiencies in supply chain management that were leading to increased procurement costs and delayed deliveries. By analyzing data, negotiating contracts with vendors, and implementing process improvements, we streamlined supply chain operations, reduced procurement costs by 20%, and improved delivery timelines. This solution not only saved the organization substantial resources but also enhanced operational efficiency and customer satisfaction.

28. How do you incorporate feedback into your problem-solving process?

Incorporating feedback into the problem-solving process is essential for continuous improvement. I actively seek feedback from stakeholders, team members, and subject matter experts throughout the problem-solving journey. I analyze feedback to identify areas for enhancement, consider alternative perspectives and solutions, and iteratively refine strategies based on input received. Regular feedback loops ensure that solutions are well-informed, aligned with stakeholder expectations, and optimized for effectiveness.

29. What is the most unconventional problem-solving method you’ve successfully used?

In a project where traditional problem-solving methods were insufficient, I applied design thinking principles to generate innovative solutions. By conducting empathy interviews, brainstorming sessions, and prototyping ideas, we identified creative solutions that addressed user needs more effectively. This unconventional approach led to breakthrough solutions that significantly improved user experience and product performance.

30. How do you ensure that your solution aligns with the overall goals of your organization?

Aligning solutions with organizational goals involves understanding strategic objectives, engaging stakeholders, and evaluating solutions based on their impact on key performance indicators. I ensure alignment by conducting thorough analyses, seeking input from relevant stakeholders, and assessing how proposed solutions contribute to organizational priorities, such as cost reduction, revenue growth, customer satisfaction, or market expansion. Regular alignment checks and feedback loops help validate that solutions remain aligned with overarching goals.

31. Can you describe a problem you solved that had a positive impact on your team’s performance?

In a team performance improvement project, I identified communication barriers and lack of collaboration as key challenges impacting productivity. I implemented team-building activities, fostered a culture of open communication and collaboration, and facilitated regular feedback sessions. These interventions resulted in improved teamwork, enhanced morale, and increased productivity, leading to measurable performance improvements and positive outcomes for the team and organization.

32. How do you decide when to tackle a problem on your own or to seek help?

I assess the complexity, urgency, and impact of the problem to determine whether I can address it independently or if seeking help is necessary. If the problem requires specialized expertise, collaboration, or resources beyond my capabilities, I proactively seek assistance to ensure efficient and effective problem-solving. I prioritize transparency and open communication to foster a collaborative problem-solving approach.

33. How do you keep track of multiple problems that need to be solved at the same time?

To manage multiple problems simultaneously, I utilize organization tools such as task lists, project management software, and prioritization techniques. I categorize problems based on urgency, importance, and dependencies, establish clear timelines and milestones for each, and regularly review progress to adjust priorities as needed. Effective delegation, communication, and collaboration with team members also contribute to successful management of multiple problem-solving efforts.

34. Can you explain how you evaluate potential solutions to a problem?

When evaluating potential solutions, I consider factors such as feasibility, effectiveness, cost, impact, and alignment with objectives. I conduct thorough analyses, gather relevant data and insights, assess risks and benefits, and compare alternative solutions based on predefined criteria. I prioritize solutions that are practical, sustainable, and aligned with strategic goals, and I seek input from stakeholders and subject matter experts to ensure comprehensive evaluation and informed decision-making.

35. How do you deal with resistance when implementing a solution?

Addressing resistance during solution implementation requires effective communication, stakeholder engagement, and change management strategies. I proactively communicate the rationale behind the solution, its benefits, and the expected outcomes to gain buy-in and mitigate resistance. I listen to concerns, address objections, and involve stakeholders in the decision-making process to foster ownership and collaboration. I also provide support, training, and feedback mechanisms to facilitate smooth adoption and implementation of the solution.

36. Can you describe a time when you had to solve a problem outside of your area of expertise?

In a cross-functional project, I encountered a technical issue that required expertise beyond my domain. Recognizing the challenge, I collaborated with experts from relevant departments, sought their insights, and leveraged their knowledge to understand the problem thoroughly. By facilitating interdisciplinary discussions, conducting research, and learning from experts, I gained the necessary understanding to contribute effectively to problem-solving and drive successful outcomes for the project.

37. How do you ensure that your problem-solving efforts are inclusive and consider diverse perspectives?

Inclusive problem-solving involves valuing diverse perspectives, fostering open dialogue, and creating an environment where everyone's input is respected and considered. I actively seek input from team members with varying backgrounds, experiences, and expertise, encourage brainstorming sessions, and facilitate discussions that promote diverse viewpoints. I promote inclusivity by practicing active listening, empathy, and respect for different opinions, ensuring that solutions are comprehensive, innovative, and reflective of diverse perspectives.

38. Can you share a situation where you had to use emotional intelligence in problem-solving?

During a project with tight deadlines and high stakes, I encountered interpersonal conflicts among team members that were impacting productivity and morale. Recognizing the importance of emotional intelligence, I employed active listening, empathy, and conflict resolution skills to understand each team member's perspectives, address underlying issues, and foster a collaborative and supportive environment. By acknowledging emotions, managing conflicts constructively, and promoting effective communication, I successfully mitigated tensions, restored team cohesion, and achieved project objectives.

39. How do you manage stress and pressure when solving critical problems?

Managing stress and pressure during critical problem-solving involves prioritization, time management, self-care, and resilience-building strategies. I prioritize tasks based on urgency and importance, break down complex problems into manageable steps, and set realistic expectations and timelines. I practice stress-reduction techniques such as mindfulness, exercise, and time management, seek support from colleagues and mentors, and maintain a positive mindset to navigate challenges effectively and maintain focus on problem-solving objectives.

40. Can you explain a time when you had to prioritize long-term solutions over quick fixes?

In a previous project, we encountered recurring quality issues that required immediate attention. While quick fixes could address the immediate symptoms, I advocated for a comprehensive root cause analysis to identify underlying systemic issues. By prioritizing long-term solutions, we implemented process improvements, updated quality control measures, and provided training to team members. This approach not only resolved the immediate issues but also prevented future occurrences, leading to sustainable improvements and long-term success.

41. How do you adapt your problem-solving approach based on the audience or stakeholders involved?

Adapting problem-solving approaches involves understanding stakeholders' preferences, priorities, and communication styles. For technical audiences, I focus on data-driven analyses, detailed reports, and solution feasibility. When engaging non-technical stakeholders, I emphasize clear explanations, visual aids, and real-world examples to ensure understanding and alignment. I tailor communication channels, formats, and frequency to suit stakeholders' preferences, fostering collaboration, buy-in, and successful problem resolution.

42. Can you discuss a time when you had to use negotiation skills in problem-solving?

During a project negotiation, conflicting interests arose between departments regarding resource allocation and project timelines. To resolve the impasse, I facilitated negotiations by identifying common goals, exploring compromise options, and advocating for win-win solutions. Using active listening, empathy, and persuasive communication, I bridged differences, built consensus, and reached agreements that satisfied all parties' needs. This collaborative approach not only resolved conflicts but also strengthened relationships and promoted cooperation.

43. How do you incorporate new technologies or methodologies into your problem-solving toolkit?

Incorporating new technologies and methodologies into problem-solving requires continuous learning, experimentation, and adaptation. I stay updated on industry trends, attend training sessions, and seek opportunities to apply emerging technologies such as data analytics, automation, and AI in problem-solving. I collaborate with experts, conduct pilot projects, and evaluate results to assess the effectiveness and feasibility of new approaches. By embracing innovation, I enhance problem-solving capabilities, drive efficiencies, and deliver value to organizations.

44. Can you describe a time when your problem-solving led to innovation within your organization?

In a project focused on streamlining operations, I identified inefficiencies in existing processes and proposed innovative solutions leveraging digital tools and automation. By collaborating with cross-functional teams, conducting workflow analyses, and piloting new technologies, we implemented streamlined workflows, reduced manual tasks, and improved efficiency. This innovation not only optimized operations but also enhanced employee productivity, customer satisfaction, and organizational competitiveness.

45. How do you ensure that your solutions are sustainable and environmentally friendly?

Ensuring sustainable and environmentally friendly solutions involves considering ecological impacts, resource conservation, and long-term viability. I prioritize sustainable practices such as waste reduction, energy efficiency, and eco-friendly materials in solution design. I collaborate with sustainability experts, conduct lifecycle assessments, and integrate environmental considerations into decision-making processes. By promoting green initiatives, I contribute to environmental stewardship, corporate social responsibility, and positive societal impact through problem-solving efforts.

46. Can you share an example of a cross-functional problem you solved?

In a previous role, we faced a cross-functional challenge related to customer retention. The marketing team identified a decline in customer engagement, while the sales team noticed an increase in churn rates. Collaborating with both teams, I led a comprehensive analysis to identify root causes. We discovered that misaligned messaging between marketing campaigns and sales interactions was confusing customers. By implementing a unified communication strategy, aligning marketing messages with sales processes, and enhancing customer touchpoints, we improved customer satisfaction, reduced churn rates, and increased revenue.

47. How do you mentor others in developing their problem-solving skills?

Mentoring others in problem-solving involves providing guidance, encouragement, and opportunities for skill development. I mentor by sharing my problem-solving experiences, providing frameworks, and encouraging critical thinking. I offer constructive feedback, challenge assumptions, and encourage diverse perspectives to foster creativity and innovation. I also create learning opportunities such as workshops, case studies, and collaborative projects to help mentees apply problem-solving techniques in real-world scenarios and enhance their skills over time.

48. Can you describe a situation where you had to overcome significant obstacles to solve a problem?

In a complex project, we encountered unforeseen technical challenges that threatened project timelines and deliverables. Despite initial setbacks and resource constraints, I led the team in brainstorming sessions, leveraging expertise from various departments, and exploring alternative solutions. We overcame obstacles by fostering collaboration, adopting agile methodologies, and prioritizing problem-solving efforts. Through perseverance, resilience, and adaptability, we successfully resolved technical issues, met project milestones, and delivered high-quality outcomes.

49. How do you balance logic and intuition in your problem-solving process?

Balancing logic and intuition in problem-solving involves leveraging analytical thinking and creative insights. I start by gathering data, analyzing facts, and applying logical frameworks to understand the problem's scope and complexity. Then, I tap into intuition by brainstorming ideas, exploring unconventional approaches, and considering potential outcomes from different perspectives. I combine rational decision-making with gut instincts, test hypotheses, and validate solutions to ensure effectiveness and innovation in problem-solving.

50. Can you discuss a time when you had to revisit and revise a solution based on new information or outcomes?

In a strategic initiative, we implemented a new process to streamline operations and improve efficiency. However, after implementation, we noticed unexpected challenges and suboptimal results. Recognizing the need for continuous improvement, I initiated a review process, gathered feedback from stakeholders, and conducted performance evaluations. Based on new insights and outcomes, we revisited the initial solution, identified areas for enhancement, and revised the process to address gaps and achieve desired outcomes effectively. This iterative approach ensured that our solutions remained adaptive, responsive, and aligned with evolving needs and objectives.

Get More Interviews, Faster

Huntr streamlines your job search. Instantly craft tailored resumes and cover letters, fill out application forms with a single click, effortlessly keep your job hunt organized, and much more...

or learn more

Next-Generation Job Tailored Resumes

Huntr provides the most advanced job <> resume matching system in the world. Helping you match not only keywords, but responsibilities and qualifications from a job, into your resume.

Job Keyword Extractor + Resume AI Integration

Huntr extracts keywords from job descriptions and helps you integrate them into your resume using the power of AI.

Application Autofill

Save hours of mindless form filling. Use our chrome extension to fill application forms with a single click.

Move beyond basic, bare-bones job trackers. Elevate your search with Huntr's all-in-one, feature-rich management platform.

Perfectly tailored cover letters, in seconds! Our cover letter generator blends your unique background with the job's specific requirements, resulting in unique, standout cover letters.

Huntr checks your resume for spelling, length, impactful use of metrics, repetition and more, ensuring your resume gets noticed by employers.

Gorgeous Resume Templates

Stand out with one of 7 designer-grade templates. Whether you're a creative spirit or a corporate professional, our range of templates caters to every career aspiration.

Personal Job Search CRM

The ultimate companion for managing your professional job-search contacts and organizing your job search outreach.

problem solving interview riddles

problem solving interview riddles

6 Brain-teasing Google Interview Questions and How to Answer Them

' src=

Not long ago we have written a blog post with most common IBM Interview Questions and how to answer them. This time we are willing to share you most common Google interview questions since this tech giant is one of one of the world’s most valuable brands that makes sure to hire the right people who can solve complex problems while working in a challenging environment. 

Google is popular for asking brain-teaser questions to test the mental sharpness of potential candidates. These interview questions might don’t have specific answers, but they can certainly help companies gauge the ability of a candidate to work under pressure. 

If you’re passionate about working at Google, be sure to prepare yourself for hypothetical questions that don’t necessarily have a right or wrong answer. This makes Google interview questions much different when compared to any other interview questions. It’s not about knowing the answers, rather you need to think broadly and quantitatively to show how good you’re at tackling problems on the spot. You can’t be certain about the interview questions. So, all you have to do is to practice and improve your analytical skills. 

Business Insider revealed some of the questions Google had asked candidates over the years. Let’s take a look at 6 of Google’s brain teaser interview questions and how you should answer them: 

1. How many golf balls can be easily fitted inside the school bus?

This is one of those confoundedly difficult questions. Even if you know the width and length of a standard school bus and the volume of a golf ball, solving this kind of question during a job interview can be difficult. Let’s see how we can answer it mathematically: 

Simulate your next interview

Prepare for the questions that are really going to be asked in your next interview.

Let’s assume a standard school bus is 6ft high and 20ft long, which means 960 cubic feet or 1.6 million cubic inches. If the volume of a golf ball is 2.5 cubic inches, divide 2.5 into 1.6 million. The answer is 660,000 which is the number of golf balls you can fit inside the school bus. Here is another similar question: Estimate the number of tennis balls that can fit into a plane.

A question like this can be asked to evaluate your approach towards solving a problem. It’s not important to come up with an exact answer, but your response should be based on the right thought process. 

2. Explain a database in three sentences to an 8-year-old.

Companies, whether small or large, need people with great communication skills no matter the job role. This question is a good one to test the ability of an applicant to communicate complex ideas. The answer isn’t something specific. All you have to do is simplify the concept. For example, ‘a database is a machine that is good at remembering plenty of information’. Make sure to check out Apple’s question with a similar subject.

Start practicing Google Interview Questions now!

3. you have eight balls all of the same sizes, seven of them weigh the same, and one of them weighs slightly more. how can you find the ball that is heavier by using a balance and only two weightings.

Here is a simple solution! Take 6 balls and put 3 on each side of the scale. It will tell you whether or not the one that weighs slightly heavier than others is in the group of 6. If the weigh on both sides is equal, you know the heavier ball is in the remaining two. So, you weigh the remaining two to solve the puzzle. If the heavy ball is in the group of 6, pick 3 balls that are heavier than the 3 on the other side of the scale. Now, pick 2 of them and put them on the scale. If they are of the same weight, the heavy ball is the one you sat to the side. 

4. You have a 3-gallon jug and 5-gallon jug, how do you measure out exactly 4 gallons? 

There’s no way you can measure out exactly 4 gallons in a 3-gallon jug. So, you need the 5-gallon jug to solve the problem. First, fill the 3-gallon jug and pour it into the 5-gallon jug. Now, again fill the 3-jug gallon and pour it into 5-gallon jug. Since the 5-gallon jug already has 3 gallons in it, only 2 more gallons can be poured into it. Now you have 1 gallon left in the 3-gallon jug. 

We’re so close to solving this problem. Empty the 5-gallon jug and pour 1 gallon left in the 3-gallon jug into it. Fill the 3-gallon jug one last time and pour it into the 5-gallon jug. Congratulation! You have measured out exactly 4 gallons. 

5. How much should you charge to wash all the windows in Seattle?

Some questions look difficult but are surprisingly simple to answer. In the case of this question, you don’t have to and you can’t count all the windows in Seattle to come up with a quote. So, the trick is to find an easier answer. Let’s say you would charge $10 per window. Problem solved!

6. Why are manhole covers round? 

This question has been around for many years. Large organizations like Google and Microsoft have asked this one to test the ability of a candidate to think on their feet. 

Why aren’t manhole covers square or triangle? There is more than one reason why manhole covers are round. First, it’s the best shape to resist the compression of the earth around it. Second, round covers are easy to manufacture and transport. Third, round covers are easy to slip into place. One of the leading reason manhole covers are round is that they can’t fall through a circular opening. 

Bottom line 

It’s critical to use a well-thought-out interview strategy and determine what kind of questions you need to practice, especially when you’re targeting a specific company. Most of the leading tech companies ask brain teaser questions while interviewing candidates for different positions. So, make sure to practice solving tricky questions. 

More than two million people apply to land a job at Google every year. However, getting a job at Google is almost ten times harder than it is to get into Harvard. To improve your chances of acing the Google interview, make your interview preparation process more efficient and smart. 

Start Practicing Interview Questions Now!

According to Glassdoor, every corporate job attracts 250 applicants on average. Gain an edge over these other applicants by learning from previous interviewees.

If you found Google interview questions interesting then make sure to check out Microsoft hiring process next.

' src=

  • 41 of Google’s Toughest Interview Questions

problem solving interview riddles

Featured in:

problem solving interview riddles

Google is one of the best companies to work for in the world. Google employees have perks that are unheard of in other companies.

They get to enjoy free gourmet cafeterias, massage rooms, free health checkups, nap pods and so on. If they want to take a break, Google employees can relax by playing billiards, swimming in the company pool, enjoying some video games or giving the rock climbing wall a try.

Consequently, the company gets a lot of applications from job seekers.

To help it find the best employees from all these applicants, Google is known to have a very rigorous interviewing process, with difficult, brainteaser questions that require you to think outside the box.

These questions are thrown in between practical interview questions with the aim of testing how prospective employees act under pressure.

Many of these questions do not have any definite answer. Instead, they are meant to provide the interviewer with a glimpse into the interviewee’s problem solving skills and thought processes.

Below are the 41 toughest questions you might have to answer if you ever find yourself invited to an interview at Google.

1. How many golf balls can be fitted inside a school bus?

The aim of this question is to find out your approach to solving problems. Therefore, what matters is not the exact answer, but rather the thought process you follow to arrive at the answer.

To answer this question, you can guestimate the dimensions of an average school bus and then use these dimensions to calculate the volume of the bus.

From there, guestimate the volume of a golf ball and then divide the volume of the school bus by the volume of a golf ball to find out the number of golf balls that can fit inside the bus.

By explaining this process, you will have provided the interviewer with an insight into how you solve problems.

2. You need to confirm that your friend Bob has your correct phone number…

However, you cannot ask Bob directly. You have to write down the question on a piece of paper and give it to Eve, who will pass the paper to Bob and then pass back Bob’s answer to you. What should you write on the paper to ensure that Bob gets your message while making it impossible for Eve to find out your phone number?

This question is meant to test your creativity and your ability to think outside the box. A simple answer would be to ask Bob to give you a call.

If he makes the call, you will be certain that he has your correct number. If he doesn’t, then that means that he doesn’t have your number.

If you want to give an even more geeky answer, you can ask Bob to write down the sum of all the digits that make up your phone number. This is known as a check-sum.

If Bob doesn’t have your correct number, his answer will not be equal to the sum of all the digits of your number. In addition, since Bob will only write down the sum, it will be impossible for Eve to find out your number.

3. If you were asked to wash all the windows in Seattle, how much would you charge?

This is a trick question meant to test whether you can find simple solutions to complex problems. Instead of trying to guess the number of windows in Seattle, you can answer the question simply by stating the price you would charge per window, for example, $8 per window.

4. You are given eight balls which are equal in size…

Seven of these balls have an equal weight, while one ball is slightly heavier. You have also been provided with a weighing balance. How do you find the heavier ball in only two weighings?

The aim of this question is to test your ability to be creative in the face of limited resources. In this case, the answer is to first take 6 balls and place them on the weighing balance, three on each side.

If the balls balance each other, then you know that the heavier ball is not part of this group, and is therefore one of the remaining two balls. You can then put these two remaining balls on the balance to determine the heavier one. If the heavier ball is part of the six balls, one side of the balance will be heavier, and you will have narrowed down to the three balls on the heavier side.

You can now pick two balls from this group and place them on the balance. If the heavier ball is among the two balls on the balance, you will know which one it is. If the two balls balance each other, then the third ball is the heavier one.

5. Come up with an evacuation plan for this city.

This is another question that is meant to provide the interviewer with a glimpse into your thought processes while solving problems.

The best response to this question would be to first ask the kind of disaster you are planning for.

6. How many piano tuners are there in Chicago?

Once again, this question is meant to test your thought processes when solving problems. What matters is not giving an exact answer, but rather explaining the thought process that led you to the answer.

To answer this question, you should start by making a guestimate of the total population of Chicago.

From the total population, you can come up with an estimate of the number of households within the city.

Next you need to make an estimate of the number of households that own a piano.

From there, estimate the number of times that a piano is tuned each year. From this, you can come up with an estimate of the number of piano tunings that are done each year.

From there, make an estimate of the average time it takes for a piano tuner to tune the piano.

Assuming that each piano tuner works 8 hours a day for 5 days a week, it is possible to calculate the number of piano tuners within the city.

7. There are four people stranded at night on one side of a cliff who need to go across a rickety rope bridge…

Unfortunately, the group has only one flashlight, and the batteries on the flashlight can only keep it powered for 17 minutes. Crossing the bridge without a flashlight is too dangerous, and the bridge can only support two people at a time. The four people have different speeds of going across the bridge. The first person can cross the bridge within a minute. The second one needs two minutes to cross the bridge. The third person needs 5 minutes to cross the bridge, while the last person needs 10 minutes to go across the bridge. How can the group make it across the bridge before the batteries on the flashlight die?

This question is simply a riddle meant to test your creativity. The correct answer is to have person one and two cross the bridge together, taking two minutes.

Person one goes back with the flashlight, with three minutes spent so far. Person three and four then cross together in ten minutes, taking the total time spent to 13 minutes.

Person two would then go back with the flashlight, taking total time spent to 15 minutes.

Person one and two would then cross the bridge together in the two minutes remaining before the flashlight goes off.

8. Assume you are the captain of a pirate ship and you have just stolen some loot from another ship…

As the captain, you get to decide how the loot will be divided. However, the crew will have to vote on your decision. If more than half of the crew vote against your plan, you will be killed. How will you recommend that the loot be divided in such a way that you stay alive and still get a good share of the loot?

This question aims to test your ability to come up with solutions for problems that deal with consensus in computing. This question is usually directed to engineering managers.

The answer to this question is to divide the loot evenly between 51% of the crew.

9. Your body is reduced in size to the height of a nickel and then you are thrown into an empty blender. The blender will be turned on in 60 seconds. What will you do?

This is another question that is meant to test your creativity. Like many of the questions, there is no definite answer to this question.

You could try and tamper with the electric motor before the blades start moving.

10. Explain a database to your 8 year old nephew in three sentences

The aim of this question is to test your understanding of complex ideas and your ability to explain these ideas in simple language.

A simple answer to this question would be: “ A database is a machine where you can store lots of information about something. You can then go to this machine whenever you want to remember that information.”

11. How much money does Google make from Gmail ads daily?

This question is meant to test your knowledge of Google’s different revenue systems as well as your analytical skills. What matters here is not the exact answer, but your explanation of how you arrive at your answer.

For instance, you might start your answer by stating the number of ads that Google places in each opened Gmail email. You can then estimate the average amount advertisers pay for each click on the ads.

F rom this, you can calculate the amount of money Google makes daily from Gmail ads by multiplying the number of Gmail users by the average number of emails each user receives each day by the average rate paid per clicked ad, divided by the multiple of email open rate and click through rate.

12. What is the weight of the empire state building?

This is another question that is meant to test your thought processes.

Once again, the aim here is not to give the exact weight of the empire state building. Unless you had anticipated the question and googled the question, it would be impossible to know the exact weight.

What you want to do in case this question is asked is to explain how you would go about calculating the weight of the building.

13. If the time on a wall clock is 3:15, what is the angle between the minute hand and the hour hand?

This is a trick question that whose aim is to test your attention to detail. Many people are likely to say the angle is zero since 3 o’clock and 15 minutes past the hour are both represented by the number 3 on a clock.

However, you should keep in mind that the hour hand gradually moves to the next number as the hour progresses. Therefore, while the minute hand will be pointing at 3, the hour hand will be a quarter way between 3 and 4.

Since there are 60 minutes in one revolution of the minute hand, we can say that each minute is equal to 6 degrees.

Therefore, the angle between two numbers on the clock is 30 degrees. Since the hour hand is ¼ way between 3 and 4, while the minute hand is at 3, the angle between them will be ¼ of 30 degrees, which is 7.4 degrees.

14. What should we have for dinner tonight?

Yes, this is actually a question that has been asked during a Google interview. The aim of this question is to test your ability to be a leader.

Therefore, you should give a definite answer, instead of saying something like “Depends on what you like” or “what do you have in mind?”

15. What is the significance of “dead beef”?

This question is usually asked to computer engineers with the aim of testing your knowledge of the industry.

Don’t respond by saying that beef is always dead.

Instead, you should be aware that DEADBEEF is a computing term that refers to a hexa-decimal value that was used to debug mainframe computers.

16. Is your IQ more than 130?

This is a trick question that is meant to assess your intellectual humility and how you view yourself.

Avoid answering this question with a yes even if you know your IQ is above 130.

People who take IQ tests and remember the results are likely to be insecure and prone to self-aggrandizement.

17. There are months with 30 days and others with 31 days. How many months have 28 days?

This is another trick question. Don’t be tempted to give “February” as your answer, because all months have 28 days or more.

18. You are presented with six glasses arranged in a row…

The first three glasses are empty, while the next three glasses are full of juice. You are required to arrange these glasses such that empty glasses alternate with full glasses. You are only allowed to move one glass. How do you do it?

This question is meant to test your creativity. The correct answer is to take the fifth glass, which is full, and pour its contents into the second glass.

19. You want to bring your dog to the office but one of your colleagues is allergic to dogs. What will you do?

The aim of this question is to assess how you would deal with potentially conflicting situations between you and your colleagues. A good answer would be to either leave your dog at home or ask to work remotely.

20. Between a flower shop and a funeral home, which has greater advertising potential?

Advertising is one of Google’s main sources of revenue, so this question tests your understanding of the company’s business model.

When answering this question, you should explain why you think one has greater potential than the other.

For instance, you could say that the funeral home has greater advertising potential because it has a well-defined target audience and buyer intent keywords.

21. What do you think the term being Googley means?

This question is meant to test whether you have a good understanding of company culture at Google. Google looks for crazy but innovative nerds. T

herefore, being Googley would mean being a crazy nerd who would fit at Google.

22. If you got this job, what prank would you pull on your manager?

This is another question that is meant to test your personality and your knowledge of company culture.

Most Googlers are a fun lot to work around, therefore the company wants to know if you are the kind of person who would fit in such a group.

However, I don’t think pulling a prank on your manager is such a good idea.

23. You have a piece of paper that is 1mm thick. How many times would you need to fold the piece of paper in half for it to be high enough to reach the moon?

This is a trick question that is meant to test how you react when faced with an unexpected question, as well as to assess how well you understand situations that involve exponential growth.

While it would be really difficult to come up with the correct answer during an interview, the answer is 42. Folding a 1mm thick piece of paper in half 42 times will make the paper 4,398,046 km in height, which is more than the distance from the earth to the moon.

24. Why are manhole covers round?

This is a pretty common question in Google interviews, and it’s surprising that many people do not know the answer. Manhole covers are round because a circle is the only shape that cannot fall through itself.

25. If you had to remove ads from YouTube, how would you continue making money from the platform?

This question is meant to test your knowledge of monetization options available in the digital space. Here, you can talk about options like creating a premium version of the platform, or charging content creators to post on the platform.

26. How would you explain the internet to someone who has absolutely no clue what it is?

This question is meant to test your knowledge of the internet – the chief space in which Google operates – as well as your communication skills.

27. You are at a party where there are 10 people, including you and a friend…

Your friend proposes a wager where you get $1 for every person in the party who shares their birthday with you, while he gets $2 for every person who has a different birthday from you. Would you take up the wager?

Unless there’s a seasonal increase in births around the month you were born, you should avoid this wager, since the probability of someone sharing the same birthday as you is 1/365.

28. What would you spend your time doing if working was not necessary?

If you did not have to work to earn a living, you would definitely spend time doing the things you love. This question is meant to give the interviewer some insight into your passions.

29. What would I learn about you by opening your browser history?

This question is similar to the previous question in that it tries to understand your passions and interests. Of course, most people spend the highest percentage of their time online on things that interest them.

30. Assume an advertiser makes $0.10 every time an ad is clicked. Only 20% of visitors click on this ad. How many people does the advertiser need to visit the site in order for him to make $20?

Such a question might seem easy, but it can be pretty confusing when you are sitting nervously before a panel. The answer to this question is 1000 people.

31. Which is your favorite product from Google, and how would you make it better?

This question is meant to test your understanding of Google’s products. If you are going for an interview at Google, you definitely need to know how most of their products work.

32. How many ways are there for finding a needle in a haystack?

The aim of this question is to test your ability to be creative and to think outside the box. The answers to this question are as many as you can think of.

You might opt to use a metal detector, you might choose to burn the haystack at a temperature that leaves the needle intact while reducing the hay into ashes, or you might decide to painstakingly go over each straw of hay, though this might take you several months to find the needle.

33. What changes do you think will happen in the digital advertising space in the next 3 years?

Advertising is one of Google’s core sources of revenue, and therefore you need to have a good understanding of digital advertising if you want to work at Google.

This question is meant to test your understanding of digital advertising and to test whether you are up to date with the trends in the industry.

34. How would you describe AdWords to a 7 year old?

This question is meant to test your understanding of how AdWords works and your ability to explain a complex feature in simple language.

35. How many haircuts are made annually in the United States?

Yes, this is another of those seemingly impossible questions that will pop up during an interview at Google. Here, the aim is to find out your thought processes, rather than the actual number of haircuts that are made in the US every year.

Here you can make a guestimate by making some assumptions.

For instance, you might assume that the average American gets a haircut every two months, which is a total of about six times every year. Multiplying this by America’s population will give you a ball park figure of the number of the number of haircuts that happen in the country every year.

36. If you had to explain the importance of HTML to Sergey Brin and then to your grandmother, how would you do it?

The aim of this question is to test your communication skills. It tests your ability to explain a complex concept both in technical terms and in layman’s language.

When explaining to Sergey Brin, you would use technical language because he is a computer scientist.

However, when explaining to your grandma, you would have to use layman’s language because she probably would not understand the technical terms.

37. Do you prefer learning or earning?

This question tries to assess whether you care more about improving your skills or making money. If you want to work at Google, you have to show a commitment to continuous improvement of your skills.

However, this doesn’t mean that you should pay zero attention to your income. A good answer would be: “I prefer a position that allows me to work (earn some money) while learning new things.

38. If you had $10 million in your account, how would you spend it?

Ever heard of the saying ‘show me where you spend your money and I will tell you what your values are’? This question is meant to give the interview an insight into what your greatest priorities are.

39. If you are given a spacecraft and $1 billion, how would you solve mankind’s biggest problem?

Google X’s moonshot projects are aimed at using technology to solve some of the world’s biggest problems. If you answer this question properly, you might end up getting sent to Google X’s top secret research lab in California.

40. How many times do the hands of a clock overlap each day?

This question is asked to simply present you with an unexpected question and see how you react under pressure. Don’t stare at the interviewer in bewilderment. The hands of a clock overlap 22 times every day.

41. Would it be a good idea for Google to start charging Gmail users $1 per month?

The aim of this question is to test your knowledge of the dynamics of monetizing a service like Gmail. Charging users $1 per month would probably not be a very good idea, since the cost of introducing and processing the payments would keep the venture from being profitable.

The fee would also lead to a decrease in the number of Gmail users.

WRAPPING UP

These are some of the tough questions that might get thrown your way in case you find yourself interviewing at Google.

Knowing the kind of questions that are asked will help you prepare for all kinds of scenarios so you don’t find yourself bewildered once you are asked to state the weight of the empire state building.

However, many of the big companies are abandoning these kinds of questions because they are not accurate predictors of job performance, and hopefully, Google will also have abandoned them by the time you get invited for an interview.

problem solving interview riddles

Comments are closed.

Related posts

17 of the Weirdest Interview Questions Google and Other Big Companies Ask to Identify Top Talent

Job interviews look alike. There is the introduction, the questioning, the negotiations. There are …

3 Steps to Apply for a Job When You Don’t Meet the Requirements

While job hunting, you see a vacancy that looks like a good fit for you. You get interested and take …

15 Funny Interview Questions

Attending an interview for a new job can be nerve-wracking and you need to know how to handle the …

408,000 + job opportunities

problem solving interview riddles

Not yet a member? Sign Up

join cleverism

Find your dream job. Get on promotion fasstrack and increase tour lifetime salary.

Post your jobs & get access to millions of ambitious, well-educated talents that are going the extra mile.

First name*

Company name*

Company Website*

E-mail (work)*

Login or Register

Password reset instructions will be sent to your E-mail.

Ace This Interview Puzzle: A Fun Riddle

' src=

This video presents a fun interview riddle from Math StackExchange. The riddle is designed to test problem-solving skills and logical thinking. The video provides a step-by-step solution to the riddle, making it a valuable resource for anyone preparing for job interviews.

Imagine you have two identical-looking bottles. One bottle contains water, and the other contains a deadly poison. You have two glasses, and you need to figure out which bottle contains the poison. You are allowed to take a single drop from each bottle and pour it into a glass. You can then observe the glasses to determine which bottle contains the poison.

The Solution

Here’s how to solve this riddle:

  • Take a single drop from the first bottle and pour it into the first glass.
  • Take a single drop from the second bottle and pour it into the second glass.
  • Observe the glasses. The glass containing the poison will have a different appearance than the glass containing water. For example, the poison might cause a color change, a chemical reaction, or a change in the liquid’s viscosity.

Why This Riddle Works

This riddle is a classic example of a logic puzzle. It requires you to think creatively and apply basic scientific principles. The key to solving this riddle is to understand that even a single drop of poison can cause a noticeable change in the liquid’s appearance.

Benefits of Solving Interview Puzzles

Solving interview puzzles can be beneficial for several reasons:

  • Demonstrate problem-solving skills: Interview puzzles assess your ability to think critically and find creative solutions to problems.
  • Showcase your analytical skills: By breaking down the problem into smaller steps, you demonstrate your analytical thinking abilities.
  • Highlight your logical reasoning: Interview puzzles often require logical deduction and reasoning skills, which are essential in many roles.
  • Engage your interviewer: Solving a puzzle can be a fun and engaging way to stand out from other candidates.

Tips for Tackling Interview Puzzles

Here are some tips for tackling interview puzzles effectively:

  • Don’t panic: Take a deep breath and approach the problem calmly.
  • Clarify the problem: Make sure you understand the problem’s parameters and any constraints.
  • Think out loud: Explain your thought process to the interviewer. This demonstrates your problem-solving approach.
  • Don’t be afraid to ask questions: If you’re unsure about something, ask for clarification.
  • Focus on the process: The interviewer is more interested in your approach than finding the right answer.

This interview puzzle is a fun and engaging way to test your logical thinking and problem-solving skills. By understanding the solution and applying the tips provided, you can confidently tackle similar puzzles in your next job interview.

Career Development

© 2024 by SchoolTube

logo

Username or Email Address

Remember Me

Forgot password?

Enter your account data and we will send you a link to reset your password.

Your password reset link appears to be invalid or expired.

Privacy policy.

To use social login you have to agree with the storage and handling of your data by this website. Privacy Policy

Add to Collection

Public collection title

Private collection title

No Collections

Here you'll find all collections you've created before.

IMAGES

  1. 20 Riddles Will Make You a Beast at Problem Solving

    problem solving interview riddles

  2. 10 Interview Puzzles || 10 Interview Riddles || commonly asked interview puzzles

    problem solving interview riddles

  3. Problem-Solving Riddles That Only Smart People Can Solve

    problem solving interview riddles

  4. Problem Solving Riddles

    problem solving interview riddles

  5. 30+ Problem Solving Riddles With Answers To Solve

    problem solving interview riddles

  6. 30+ He Landed A Job Without Interview Riddles With Answers To Solve

    problem solving interview riddles

VIDEO

  1. interview riddles 🧩#amezing #subscribers #fast #india #maths #viralshorts #like

  2. Can You Solve These Riddles?

  3. JS Problem Solving Questions 01

  4. Interview Riddles || Brain Teaser || Maths Olympiad ||

  5. Problem solving LeetCode interview questions بالعربي

  6. 10 Strategies to Improve Your Problem Solving Skills #shorts

COMMENTS

  1. 21 Puzzle Interview Questions (With Example Answers)

    Interview puzzles can encompass a variety of distinct problems, although most resemble the three main types of puzzles, which include: 1. Heaven's gate riddle question. Interviewers may present this puzzle much like a riddle or question, with a trick answer. These puzzles may assess your ability to apply logical thinking to solve the problem ...

  2. 30 Brain Teaser Interview Questions & Detailed Answers

    Answer 1: "Post" and "office". Answer 2: One that shows an indirect relationship between smoking and lung cancer, i.e "smoking causes X, X causes lung cancer". The key here is to look at "cause" as a direct relationship. Answer 3:

  3. 8 Tough Brain Teaser Interview Questions from Google, Apple and

    Question 6: You have 100 coins laying flat on a table, each with a head side and a tail side. 10 of them are heads up, 90 are tails up. You can't feel, see or in any other way find out which 10 are heads up. Your goal: split the coins into two piles so there are the same number of heads-up coins in each pile. Answer….

  4. 8 Tough Brain Teaser Interview Questions and Answers in 2024

    Brain teaser interview questions, also known as puzzle or logic questions, are designed to challenge candidates and evaluate their problem-solving, critical thinking, and creativity skills. They are usually asked in job interviews for positions that require analytical thinking or problem-solving abilities. Companies use these questions to assess a candidate's ability to respond to unexpected ...

  5. 8 Brain Teasers for Interview Insights

    Creative problem-solving; Mathematical thinking; Pros and cons of using brain teasers for interview questions. Brain teasers can offer great insight because they allow you to see a candidate's thought process and how they might problem-solve on the job. This is especially important if you are hiring for arole that requires lots of problem ...

  6. Great Brain Teaser Interview Questions (With Answers)

    Interviewers will often ask brain teaser interview questions because they will help assess your workplace skills. Answering brain teasers correctly can involve critical thinking, problem-solving, creativity, and close listening skills.For these reasons, they're commonly used during interviews to see how well and how quickly a candidate can think on his or her feet.

  7. 10 Logical Questions You Might Be Asked in an Interview

    Logical interview questions involve solving brainteasers or some type of riddle to show the interviewer your critical thinking skills, problem-solving skills and analytical skills. Logic questions for interview purposes can be used to assess these skillsets as well as gauge the way you ask for information, use resources and work under pressure.

  8. 15 brain teaser interview questions and answers

    Brain teaser interview questions are a great way to test the problem-solving skills of a candidate. They are also a great way to see how well the person can think on their feet. ... The term "brain teaser" is used to describe any puzzle, question, or riddle that requires thought and concentration to solve. Brain teasers can be classified into ...

  9. Top 20 Puzzles for Interviews

    Explanation of Interview Puzzles. An interview puzzle is a critical thinking scenario, just like brain teasers. These problems require you as an interviewee to rely on your problem-solving and lateral thinking capabilities. The primary purpose why interviewers use interview puzzles is to find out if you are a good fit for their company.

  10. Ultimate Guide to Brain Teaser Interview Questions

    Types of Interviews with Brain Teasers. You will most commonly encounter brain teaser interview questions in these industries or roles: Quantitative finance - including institutional and prop trading, hedge funds, quant trading and modeling. Consulting - including management and strategy consulting. Engineering interviews - including ...

  11. Top 20 Interview Puzzles for Software Engineers

    Prepare for your software engineering interviews with these top 20 interview puzzles. Test your problem-solving skills and get ready to impress potential employers. From coding challenges to logical riddles, these puzzles will help you sharpen your technical abilities and boost your chances of landing your dream job. Start practicing now!

  12. The Most Common Interview Puzzles and How to Answer Them

    The most common type of puzzle is the brainteaser. Brainteasers test your problem-solving and critical-thinking skills. They typically involve a mix of logic, math, and wordplay. Other common types of interview puzzles include riddles, logic problems, and pattern recognition exercises.

  13. 7 Interview Brain Teasers You Could Be Asked

    6. The "Solve This Mystery" Question. Oh, occasionally you'll be asked to go detective and solve a mystery: "A windowless room has three light bulbs. You are outside the room with three switches, each controlling one of the light bulbs.

  14. Top 20 Problem Solving Interview Questions (Example Answers Included)

    MIKE'S TIP: When you're answering this question, quantify the details. This gives your answer critical context and scale, showcasing the degree of challenge and strength of the accomplishment. That way, your answer is powerful, compelling, and, above all, thorough. 2. Describe a time where you made a mistake.

  15. Top 100 Puzzles Asked In Interviews

    4. 100 people in a circle with a gun puzzle. IgniWorld. 5. Find the total guests that are present at the party. Google. Conclusion: Puzzles are very helpful to improve logical thinking day by day. Start practicing puzzles to crack interviews.

  16. Sample Logic Puzzles Given at Job Interviews

    Sample Job Interview Logic Puzzles. Question: You have 8 jars of the same size and shape. Seven of the jars weigh 5 ounces while the eighth jar weights 6. ... The key to answering logic puzzle interview questions is not just in solving the problem, but in using sound reasoning. If you don't know an answer, try your best to explain your ...

  17. 50 Interview Questions About Problem Solving (With Answers)

    Demonstrating your ability to tackle challenges effectively can set you apart from other applicants. Here are five tips to help you showcase your problem-solving skills during an interview: 1. Use the STAR Method. Structure your responses using the Situation, Task, Action, and Result (STAR) method.

  18. 11 Interview Logic Questions To Expect (With Answers and Tips)

    Interview logic questions evaluate your critical-thinking, analytical, and problem-solving skills. Interviewers use them to determine how you ask for and process information, use resources, and respond to events decisively. Much like brainteasers, logical interview questions may not relate directly to the job.

  19. Logical Interview Questions With Sample Answers

    With riddles, the correct answer is often not the most important aspect. The interviewer asks these types of questions to assess your ability to think creatively, problem-solve with unique approaches and use your critical thinking skills to create solutions. Related: 6 Brain Teaser Interview Questions and How to Answer Them 2. Numeracy problems

  20. 6 Brain-teasing Google Interview Questions and How to Answer Them

    Take 6 balls and put 3 on each side of the scale. It will tell you whether or not the one that weighs slightly heavier than others is in the group of 6. If the weigh on both sides is equal, you know the heavier ball is in the remaining two. So, you weigh the remaining two to solve the puzzle.

  21. 41 of Google's Toughest Interview Questions

    These questions are thrown in between practical interview questions with the aim of testing how prospective employees act under pressure. Many of these questions do not have any definite answer. Instead, they are meant to provide the interviewer with a glimpse into the interviewee's problem solving skills and thought processes.

  22. Creative Problem-Solving: Ace Creative Interview Questions

    Creative problem solving is a pivotal skill that can set you apart in job interviews, especially when you're faced with questions designed to test your creative thinking. Interviewers often ask ...

  23. Ace This Interview Puzzle: A Fun Riddle

    The riddle is designed to test problem-solving skills and logical thinking. The video provides a step-by-step solution to the riddle, making it a valuable resource for anyone preparing for job interviews. ... Demonstrate problem-solving skills: Interview puzzles assess your ability to think critically and find creative solutions to problems.

  24. 60 of the Best Riddles for Kids: Can You Solve Them?

    See how many of these Easter riddles you can solve—they'll have you hunting for answers. 9 / 61. rd.com. Riddle: Feel for it ... You'll enjoy watching them problem-solving and helping them ...